Download as pdf or txt
Download as pdf or txt
You are on page 1of 107

ALL INDIA MOCK TEST 01

EXPLANATION
INDEX

1. ....................................................................................................................... 4

2. ....................................................................................................................... 5

3. ……… .............................................................................................................. 6

4. ....................................................................................................................... 7

5. ....................................................................................................................... 7

6. ....................................................................................................................... 8

7. ....................................................................................................................... 9

8. ..................................................................................................................... 10

9. ..................................................................................................................... 11

10. ..................................................................................................................... 12

11. ..................................................................................................................... 13

12. ..................................................................................................................... 13

13. ..................................................................................................................... 14

14. ..................................................................................................................... 15

15. ……… ............................................................................................................ 16

16. ..................................................................................................................... 17

17. ..................................................................................................................... 18

18. ..................................................................................................................... 18

19. ..................................................................................................................... 19

20. ..................................................................................................................... 20

21. …………. ........................................................................................................ 20

22. ..................................................................................................................... 21

23. ..................................................................................................................... 21

24. ..................................................................................................................... 22

25. ..................................................................................................................... 23

26. ..................................................................................................................... 24

27. ..................................................................................................................... 25

1
28. ..................................................................................................................... 26

29. ..................................................................................................................... 26

30. ..................................................................................................................... 27

31. ..................................................................................................................... 28

32. ..................................................................................................................... 29

33. ..................................................................................................................... 30

34. ..................................................................................................................... 31

35. ..................................................................................................................... 31

36. ..................................................................................................................... 33

37. ..................................................................................................................... 36

38. ..................................................................................................................... 37

39. ..................................................................................................................... 38

40. ..................................................................................................................... 39

41. ..................................................................................................................... 40

42. ..................................................................................................................... 40

43. ..................................................................................................................... 41

44. ..................................................................................................................... 42

45. ..................................................................................................................... 43

46. ..................................................................................................................... 45

47. ..................................................................................................................... 46

48. ..................................................................................................................... 47

49. ..................................................................................................................... 48

50. ..................................................................................................................... 49

51. ..................................................................................................................... 50

52. ..................................................................................................................... 51

53. ..................................................................................................................... 52

54. ..................................................................................................................... 54

55. ..................................................................................................................... 54

56. ..................................................................................................................... 56

57. ..................................................................................................................... 57

2
58. ..................................................................................................................... 57

59. ..................................................................................................................... 59

60. ..................................................................................................................... 61

61. ..................................................................................................................... 61

62. ..................................................................................................................... 63

63. ..................................................................................................................... 63

64. ..................................................................................................................... 65

65. ..................................................................................................................... 66

66. ..................................................................................................................... 67

67. ..................................................................................................................... 68

68. ..................................................................................................................... 69

69. ..................................................................................................................... 70

70. ..................................................................................................................... 71

71. ..................................................................................................................... 72

72. ……….. .......................................................................................................... 73

73. ..................................................................................................................... 75

74. ..................................................................................................................... 76

75. ..................................................................................................................... 78

76. ..................................................................................................................... 79

77. ..................................................................................................................... 80

78. ..................................................................................................................... 82

79. ..................................................................................................................... 83

80. ..................................................................................................................... 84

81. ..................................................................................................................... 85

82. ..................................................................................................................... 86

83. ..................................................................................................................... 87

84. ..................................................................................................................... 88

85. ..................................................................................................................... 89

86. ..................................................................................................................... 91

87. ..................................................................................................................... 92

3
88. ..................................................................................................................... 93

89. ..................................................................................................................... 94

90. ..................................................................................................................... 95

91. ..................................................................................................................... 97

92. ..................................................................................................................... 98

93. ..................................................................................................................... 99

94. ................................................................................................................... 100

95. ................................................................................................................... 101

96. ................................................................................................................... 102

97. ................................................................................................................... 103

98. ................................................................................................................... 104

99. ................................................................................................................... 105

100. ................................................................................................................... 106

1. Which of the following subjects comes under the Concurrent List of the 7 th Schedule of the Indian
Constitution ?
1. Census
2. Education
3. Marriage
4. Gambling
5. Population control and family planning
Choose the correct answer using the code given below :
(a) 1, 2, 3 and 5 only
(b) 2, 3 and 5 only
(c) 2, 4 and 5 only
(d) 1, 2, 3, 4 and 5
EXPLANATION:
The constitution specifies the distribution of powers and responsibilities between the state and central
governments. The Seventh Schedule is the most important part of the constitution in this regard. It
specifies the role and responsibilities into three lists namely, Union List, State List and Concurrent List.
The three lists have been changed since they first came into being; the Union list contained 97 subjects
and now is at 100 subjects, the state list contained 66 subjects but is now at 61 subjects, and concurrent
list contained 47 subjects but now has 52 subjects on the concurrent list.
The Indian Constitution specifies subjects like, criminal law, criminal procedure, preventive detention,
forests, Education, Marriage and divorce, protection of wild animals and birds, Bankruptcy and

4
insolvency, trade unions, industrial and labour disputes, population control and family planning etc., to
the Concurrent list.
Census comes under union list, and Betting and gambling comes under State list.
So, Option (b) is correct.

2. Consider the following statements about the Chief Election Commissioner (CEC) and other Election
Commissioners (EC) in India :
1. Article 324 of the Constitution of India mentions the qualifications for the appointment of CEC and
ECs.
2. They will be appointed based on the advice of the Council of Ministers headed by the Prime
Minister.
3. The CEC and ECs are removed from their office, like manner and on grounds as a judge of the
Supreme Court.
4. The Chief Election Commission and other Election Commissioners enjoy equal powers and receive
equal salaries.
Which of the statements given above are correct ?
(a) 1 and 2 only
(b) 1, 3 and 4 only
(c) 1, 2 and 3
(d) 3 and 4 only
EXPLANATION:
The Election Commission is a permanent and an independent body established by the Constitution of
India directly to ensure free and fair elections in the country. Article 324 of the Constitution provides that
the power of superintendence, direction and control of elections to parliament, state legislatures, the office
of president of India and the office of vice-president of India shall be vested in the election commission.
The Constitution has not prescribed the qualifications (legal, educational, administrative or judicial) of
the members of the Election Commission. So, Statement 1 is not correct.
Article 324(2) of the Constitution of India provides that the Election Commission shall consist of the Chief
Election Commissioner and such number of other Election Commissioners, if any, as the President may
from time to time fix and the appointment of the Chief Election Commissioner and other Election
Commissioners shall, subject to the provisions of any law made in that behalf by Parliament, be made by
the President.
Recently in March 2023, the Supreme Court of India (SC), in the case titled Anoop Baranwal v. Union of
India, has directed a change in the appointment mechanism of the Chief Election Commissioner (CEC)
and Election Commissioners (EC). The current procedure entails the President appointing the CEC on the
aid and advice of the Council of Ministers. The SC has ordered that until the Parliament enacts a law in
this regard, the CEC and ECs shall be appointed on the recommendation of a 3-member committee
comprising of the Prime Minister, the Chief Justice of India, and the Leader of Opposition of the Lok
Sabha. So, Statement 2 is not correct.
5
The chief Election Commissioner and other Election Commissioners are provided with the security of
tenure. He cannot be removed from his office except in same manner and on the same grounds as a judge
of the Supreme Court. In other words, he can be removed by the president on the basis of a resolution
passed to that effect by both the Houses of Parliament with special majority, either on the ground of
proved misbehaviour or incapacity. Thus, he does not hold his office till the pleasure of the president,
though he is appointed by him. So, Statement 3 is correct.
The Chief Election Commissioner and the two other Election Commissioners have equal powers and
receive equal salary, allowances and other perquisites, which are similar to those of a judge of the Supreme
Court. In case of difference of opinion amongst the Chief Election Commissioner and/or two other Election
Commissioners, the matter is decided by the Commission by majority. So, Statement 4 is correct.

3. Which of the following is treated as a ‘foreign contributions’ under Foreign Contribution Regulation
Act (FCRA) ?
1. Foreign transfers or donations exceeding Rs. 10 lakhs in a financial year
2. Earnings by a person from foreign client(s) in lieu of goods sold or services rendered.
Select the correct answer using the code given below :
(a) 1 only
(b) 2 only
(c) Both 1 and 2
(d) Neither 1 nor 2
EXPLANATION:
The FCRA regulates foreign donations and ensures that such contributions do not adversely affect
internal security. First enacted in 1976, it was amended in 2010 when a slew of new measures was
adopted to regulate foreign donations. The FCRA is applicable to all associations, groups and NGOs which
intend to receive foreign donations. It is mandatory for all such NGOs to register themselves under the
FCRA. As defined in Section 2(1)(h) of FCRA, 2010, "foreign contribution" means the donation, delivery or
transfer made by any foreign source.
 The Central Government has notified the Foreign Contribution (Regulation) Amendment Rules, 2022
to further amend the Foreign Contribution (Regulation) Rules, 2011 (FCRR, 2011). These
amendments aim to reduce compliance burden on citizens.
 As per amended Rule 6 of FCRR 2011, any person receiving Foreign Contribution (FC) more than Rs.
10 Lakh in a financial year from relatives as defined in section 2(1)(r) of the Foreign Contribution
(Regulation) Act, 2010 has to intimate to Central Government within three months of receipt of FC.
Earlier such monetary threshold limit was Rs 1 Lakh and intimation to Central Government was
required within thirty days. So, Statement 1 is correct.
Foreign contribution excludes earnings from foreign client(s) by a person in lieu of goods sold or services
rendered by it as this is a transaction of commercial nature. Any amount received, by an person from any
foreign source in India, by way of fee (including fees charged by an educational institution in India from

6
foreign student) or towards cost in lieu of goods or services rendered by such person in the ordinary course
of his business, trade or commerce whether within India or outside India or any contribution received
from an agent or a foreign source towards such fee or cost shall be excluded from the definition of foreign
contribution within the meaning of this clause. So, Statement 2 is not correct.

4. The first-ever comprehensive study on ‘System Improvement’ in the Rajya Sabha Secretariat has made
some recommendations on Parliamentary committees as well. In this context, which of the following
statement about the present system of Parliamentary Committees is not correct ?
(a) Tenure of any standing committee is one year.
(b) They submit reports to the Presiding Officers of the respective House only.
(c) They work under the direction and control of the Presiding Officer of the respective House.
(d) On specified matters, their recommendations are binding on the Parliament.
EXPLANATION:
The Constitution of India makes a mention of these committees at different places, but without making
any specific provisions regarding their composition, tenure, functions, etc. All these matters are dealt by
the rules of two Houses. Accordingly, a
parliamentary committee means a committee that:
 Appointed or elected by the House or nominated by the Speaker / Chairman
 Works under the direction of the Speaker / Chairman
 Presents its report to the House or to the Speaker / Chairman
 Has a secretariat provided by the Lok Sabha / Rajya Sabha
 The term of office of each standing committee is one year from the date of its constitution.
 It should be noted here that the recommendations of these committees are advisory in nature and
hence not binding on the Parliament.
So, Option (d) is not correct.

5. Consider the following statements regarding the Borrowing power of the Centre and the States :
1. Centre can borrow from both within and outside India, while States can borrow only within India.
2. States must obtain previous consent from the Centre before raising a loan if they already have an
outstanding loan with the Centre.
3. In practice, the Centre and the States exercise their borrowing power in accordance with the
recommendations of the Reserve Bank of India.
Which of the statements given above is/are correct ?
(a) 1 only
(b) 1 and 2 only
(c) 1 and 3 only
(d) 1, 2 and 3
EXPLANATION:

7
Article 292 of the Indian constitution provides that the Central government can borrow either within India
or outside upon the security of the Consolidated Fund of India or can give guarantees, but both within
the limits fixed by the Parliament. So far, no such law has been enacted by the Parliament.
The borrowing powers enjoyed by the state government are much less in comparison to the Central
Government. As there are various kinds of territorial and other limitations on the borrowing powers of the
state. Under Article 298 of the Indian Constitution, a state government can borrow within India (and not
abroad) upon the security of the Consolidated Fund of the State or can give guarantees, but both within
the limits fixed by the legislature of that state. So, Statement 1 is correct.
Article 293 under clause ( 3 ) of the Indian constitution provides that a State may not, without the consent
of the Government of India, raise any loan if there is still outstanding any part of a loan that has been
made to the State by the Government of India or by its predecessor Government, or in respect of which a
guarantee has been given by the Government of India or by its predecessor Government.
So, Statement 2 is correct.
In practice, the Centre has been exercising this power in accordance with the recommendations of the
Finance Commission (not the Reserve Bank of India).
The Finance Commission discusses issues related to macroeconomics, financial stability and the cost of
borrowing of the Union and States—the other issues pertaining to market-driven borrowing cost, debt
trajectories of States and recapitalization of banks with the Reserve Bank of India.
The Commission suggested that the Centre reduce the fiscal deficit to 4% of GDP by 2025-26. For states,
it recommended the fiscal deficit limit (as % of GSDP) of: (i) 4% in 2021-22, (ii) 3.5% in 2022-23, and (iii)
3% during 2023-26. If a state is unable to fully utilise the sanctioned borrowing limit as specified above
during the first four years (2021-25), it can avail the unutilised borrowing amount (calculated in rupees)
in subsequent years (within the 2021-26 period). So, Statement 3 is not correct.

6. The Constitution of India explicitly prevents which of the following from an enquiry in the court of law
?
(a) Advice tendered by the Council of Ministers to the President
(b) Matters discussed in the meeting of the Council of Ministers
(c) Assent or veto over the bills by the President
(d) None of the above
EXPLANATION:
Article 74 of the Indian constitution provides for a council of ministers with the Prime Minister at the head
to aid and advise the President in the exercise of his functions:
 There shall be a Council of Ministers with the Prime Minister at the head to aid and advise the
President who shall, in the exercise of his functions, act in accordance with such advice. However,
the President may require the Council of Ministers to reconsider such advice and the President shall
act in accordance with the advice tendered after such reconsideration.
 The advice tendered by Ministers to the President shall not be inquired into in any court.

8
 The Supreme Court has clarified the implications of Art. 74(2) in S.R. Bommai v. Union of India. No
court is concerned with what advice was tendered by the Minister to the President. The court is only
concerned with the order's validity and not with what happened in the inner councils of the President
and the Minister. An order cannot be challenged on the ground that it is not in accordance with the
advice tendered by the Minister or that it is based on no advice.
Article 74 was amended by the 42nd Constitutional Amendment Act of 1976 to the effect that the president
shall, in the exercise of his functions, act in accordance with the advice rendered by the council of
ministers. The 44th Constitutional Amendment Act of 1978 further added a provision to this article to the
effect that the president may require the council of ministers to reconsider such advice and the president
shall act in accordance with the advice tendered after such reconsideration. So, Option (a) is correct.

7. Which of the following statements regarding the Electoral Bond Scheme is not correct ?
(a) Electoral bonds can be purchased only by a citizen of India or a body incorporated in India.
(b) Registered political parties which secured not less than one percent of the votes polled in the last
General Election to the House of the People or the Legislative Assembly of the State are eligible to
receive electoral funding through the scheme.
(c) Electoral Bonds shall be encashed by an eligible Political Party only through a Bank account with
the Authorized Bank.
(d) Know Your Customer (KYC) norms are relaxed for the purchaser of Electoral bonds in order
to protect their privacy.
EXPLANATION:
The Government has notified the scheme of Electoral Bonds to cleanse the system of political funding in
the country. An electoral bond is like a financial tool used for making donations to political parties. The
general public can also issue these bonds to fund eligible political parties. The broad features of the
scheme are given below:
 Electoral Bond would be a bearer instrument in the nature of a Promissory Note and an interest free
banking instrument. A citizen of India or a body incorporated in India, registered agency or undivided
Hindu family will be eligible to purchase the bond.
 Electoral bond would be issued/purchased for any value, in multiples of ₹1,000, ₹10,000, ₹1,00,000,
₹10,00,000 and ₹1,00,00,000 from the specified branches of the State Bank of India (SBI).
 Electoral Bonds would have a life of only 15 days during which it can be used for making donation
only to the political parties registered under section 29A of the Representation of the Peoples Act, 1951
(43 of 1951) and which secured not less than one per cent of the votes polled in the last general election
to the House of the People or a Legislative Assembly.
 The bonds under the Scheme shall be available for purchase for a period of 10 days each in the months
of January, April, July and October, as may be specified by the Central Government. An additional
period of 30 days shall be specified by the Central Government in the year of the General election to
the House of People.

9
 The bond shall be encashed by an eligible political party only through a designated bank account with
the authorised bank.
The purchaser would be allowed to buy electoral bond(s) only on due fulfilment of all the extant KYC norms
and by making payment from a bank account. It will not carry the name of payee. The existing instructions
issued by the RBI regarding Know Your Customer norms of a bank’s customer would apply to the buyers
of the Electoral Bonds. The authorised bank may request for any additional information concerning the
Know Your Customer documents if it seems necessary. So, Option (d) is not correct.

8. Which of the following cases involving defamation is/are correct ?


1. It can be either civil or criminal defamation.
2. While civil defamation is clearly defined under Indian laws, criminal defamation is derived from
case laws.
3. Punishment can be simple imprisonment for a term that may extend to two years, with a fine, or
with both.
Choose the correct answer using the code given below :
(a) 3 only
(b) 1 and 2 only
(c) 1 and 3 only
(d) 1, 2 and 3
EXPLANATION:
Defamation is the act of communicating false statements about a person that injure the reputation of that
person when observed through the eyes of ordinary man. Any false and unprivileged statement published
or spoken deliberately, intentionally, knowingly with the intention to damage someone's reputation is
defamation. A man's reputation is treated as his property and such damage are punishable by law. It
could be written or verbal. Written defamation, printed or typed material or images is called as libel and
spoken defamation is called slander.
Article 19 of the Constitution grants various freedoms to its citizens. However, Article 19(2) has imposed
reasonable exemption to freedom of speech and expression granted under Article 19(1) (a). Contempt of
court, defamation and incitement to an offence are some exceptions
Under Indian law, defamation is both a criminal (punishable with imprisonment) as well as civil offence
(punishable through the award of damages). So, Statement 1 is correct.
Criminal defamation is clearly defined under Indian laws; Civil defamation is derived from case laws.
Defamation as a civil offence is punishable under the law of torts; the legality of civil defamation was at
issue in R. Rajagopal v. State of Tamil Nadu (1994). in comparison, the criminal law on defamation is
codified under the Indian Penal Code, 1860 ("IPC"). Criminal defamation is an offence that is punishable
under the Indian Penal Code 1860. Under Sections 499 and 500 of the 1860 Code, slander is a crime.
Criminal defamation is a compoundable and non-cognizable offence that is subject to bail.
So, Statement 2 is not correct.

10
Subject Criminal defamation Civil defamation

The objective is to punish the The objective is to amend the


Objective wrongdoer, to ensure that no wrong committed by the
other person does the same. person.

Branch of
Indian Penal Code Law of torts
law

Codification
Codified Uncodified
of law

Legal
Sections 499-502 of IPC No concerned provision
provisions

Judgment is based on
Judgment is based on penal
Judgment precedents and common law
provisions.
principles.

Punishment Imprisonment or fine or both Compensation

Section 501 of the IPC states that any person who prints or engraves any matter, knowing or having good
reason to believe that such matter is defamatory of any person, shall be punished with simple
imprisonment for a term which may extend to 2 years, or with fine, or with both.
So, Statement 3 is correct.

9. With reference to the North East Regional Council, consider the following statements :
1. The composition is the same as the zonal councils, and the Union Home minister acts as chairman.
2. Established by the states reorganization act of 1956.
Which of the statement given above is/are correct ?
(a) 1 only
(b) 2 only
(c) Both 1 and 2
(d) Neither 1 nor 2
EXPLANATION:
North East Regional Council functions are similar to those of the zonal councils, but the composition of
North East Regional Council is different from zonal councils. The Council comprises of Governors and
Chief Ministers of constituent States and three members to be nominated by the President as per clause
(iii) of Section 3 of the North Eastern Council (Amendment) Act, 1971.
The present composition of five Zonal Councils was set up vide Part-III of the States Re-organisation Act,
1956 of these Zonal Councils as under:
 Chairman - The Union Home Minister is the Chairman of each of these Councils.

11
 Vice Chairman - The Chief Ministers of the States included in each zone act as Vice-Chairman of the
Zonal Council for that zone by rotation, each holding office for a period of one year at a time.
 Members- Chief Minister and two other Ministers as nominated by the Governor from each of the
States and two members from Union Territories included in the zone.
The Union Home Minister is ex-Officio Chairman and the Minister of State (IC) the Department of
Development of North Eastern Region (DoNER) is the ex-Officio Vice-Chairman of the North East Council.
It has to formulate a unified and coordinated regional plan covering matters of common importance. It
has to review from time to time the measures taken by the member states for the maintenance of security
and public order in the region. So, Statement 1 is not correct.
The North Eastern States i.e. (i) Assam (ii) Arunachal Pradesh (iii) Manipur (iv) Tripura (v) Mizoram (vi)
Meghalaya and (vii) Nagaland are not included in the Zonal Councils and their special problems are looked
after by the North Eastern Council, set up under the North Eastern Council Act, 1972. The State of Sikkim
has also been included in the North Eastern Council vide North Eastern Council (Amendment) Act, 2002
notified on 23rd December 2002. Consequently, action for exclusion of Sikkim as member of Eastern
Zonal Council has been initiated by Ministry of Home Affairs. So, Statement 2 is not correct.

10. The fundamental object of the Panchayati Raj system is to ensure which among the following?
1. People's participation in development
2. Political accountability
3. Democratic decentralization
4. Financial mobilization
Select the correct answer using the code given below :
(a) 1, 2 and 3 only
(b) 2 and 4 only
(c) 1 and 3 only
(d) 1, 2, 3 and 4
EXPLANATION:
The term Panchayati Raj in India signifies the system of rural local self-government. It has been
established in all the states of India by the Acts of the state legislatures to build democracy at the grass
root level. It is entrusted with rural development.
Panchayati Raj is a system of governance in which gram panchayats are the basic units of administration.
It was constitutionalized through the 73rd Constitutional Amendment Act of 1992.
The basic objective of the Panchayati Raj system is to ensure people’s participation in development and
democratic decentralization. So, Statements 1 and 3 are correct.
The Panchayati Raj institutions are considered as local self-government meant to provide basic
infrastructure facilities, empowering weaker sections of the society and initiate the development process
at the grass-roots level of rural India.
The establishment of Panchayati Raj Institutions does not automatically lead to political accountability.

12
Though the Panchayat Raj system seeks to transfer finances and resources to the grass root government,
financial mobilization is not considered its fundamental objective.
So, Statements 2 and 4 are not correct.

11. Recently Private member bills have been introduced in the Houses of Parliament to amend the
Preamble to the Indian Constitution. In this context, consider the following statements :
1. As Preamble is not a part of the Constitution, it is neither enforceable nor justifiable in a court of
law.
2. Preamble has been amended only once so far since its inception.
3. The Legislative competence of a bill to amend the Preamble is decided by the Chairman of the
House.
Which of the statements given above is/are correct ?
(a) 1 only
(b) 1 and 3 only
(c) 2 only
(d) 2 and 3 only
EXPLANATION:
In the case of Kesvanand Bharti vs. the State of Kerala, the Supreme has held that Preamble is part of the
Constitution.
The Preamble is of extreme importance, and the Constitution should be read and interpreted in the light
of the grand and noble vision expressed in the Preamble. However, two things should be noted:-
The Preamble is neither a source of power to the legislature nor a prohibition upon the powers of the
legislature.
It is not justiciable, i.e., not enforceable in courts of Law. So, Statement 1 is not correct.
The Supreme Court has held that the preamble can be amended subject to the condition that no
amendment is done to the ‘ Basic structure’ of the constitution.
The Preamble has been amended only once by the 42nd Constitutional Amendment Act, which has added
three new words- Socialistic, Secular and Integrity- to the preamble. So, Statement 2 is correct.
If a Bill, in this case regarding the introduction of a private member bill for the amendment of the preamble,
is opposed on the ground of the legislative competence of the Council, the Council decides the same and
not the Chairman of the House. So, Statement 3 is not correct.

12. With respect to Preventive Detention in India, consider the following statements :
1. It was borrowed from the British Constitution.
2. Both the Central and State Governments can legislate laws on Preventive dentition for the
maintenance of Public order.
3. The person detained under preventive detention has the Right to Legal aid, guaranteed under
Article 21.
Which of the statements given above is/are correct ?
13
(a) 1 and 2 only
(b) 2 only
(c) 1 and 3 only
(d) 2 and 3 only
EXPLANATION:
India is one of the few nations in the world with a Constitution that provides for preventative detention in
times of peace without the protections considered necessary in other countries to protect fundamental
human rights.
The first Preventive Detention Act was passed on 26 February 1950 with the purpose of preventing anti-
national elements from carrying out acts that are hostile to the Nation’s security and defense. It was not
borrowed from the British constitution. So. Statement 1 is not correct.
According to the seventh schedule of the Indian Constitution, Preventive detention for reasons connected
with the security of a State, the maintenance of public order, or the maintenance of supplies and services
essential to the community falls under the concurrent list, where by both the Central and State
Governments can legislate laws on Preventive dentition. So, Statement 2 is correct.
Preventive Detention is an action taken by the administration on the grounds of the suspicion that some
wrong actions may be done by the person concerned, which will be prejudicial to the state.
Preventive Detention is the most contentious part of the scheme of fundamental rights in the Indian
constitutions Article 22(3) provides that if the person is arrested or detained under preventive detention
laws, then the protection against arrest and Detention provided under article22 (1) and22 (2) shall not be
available to that person.
A detainee under preventive Detention can have no right of personal liberty guaranteed by Article 19 Or
Right to Legal aid, guaranteed under Article 21. So, Statement 3 is not correct.

13. With reference to the Charter Act of 1813, consider the following statements :
1. An All-India Executive at Calcutta, having authority to make laws and regulations for all territories
under the East India Company, came into existence.
2. Under this act, the company, for the first time, accepted state responsibility in the sphere of
education.
3. The act lifted all restrictions on European immigration, and the acquisition of property in India
was lifted.
4. The Company’s monopoly over trade in India ended.
Which of the statements given above is/are correct ?
(a) 1 and 3 only
(b) 2 and 4 only
(c) 1, 2 and 4 only
(d) 2, 3 and 4 only
EXPLANATION:

14
The following are the features of Charter Act of 1813 Act:
 The Charter Act of 1833 made the Governor-General of Bengal as the Governor-General of India and
vested in him all civil and military powers. Thus, the act created, for the first time, the Government of
India having authority over the entire territorial area possessed by the British in India. Lord William
Bentick was the first Governor-General of India. So, Statement 1 is not correct.
 The Company’s shareholders were given a 10.5 per cent dividend on the revenue of India.
 The Company was to retain the possession of territories and the revenue for 20 years without prejudice
to the sovereignty of the Crown. (Thus, the constitutional position of the British territories in India
was defined explicitly for the first time).
 Powers of the Board of Control were further enlarged.
 A sum of one lakh rupees was to be set aside for the revival, promotion, and encouragement of
literature, learning, and science among the natives of India every year. (This was an important
statement from the point of the State’s responsibility for education.)
 The regulations made by the Councils of Madras, Bombay, and Calcutta were now required to be laid
before the British Parliament. The constitutional position of the British territories in India was thus
explicitly defined for the first time.
 Separate accounts were to be kept regarding commercial transactions and territorial revenues. The
power of superintendence and direction of the Board of Control was not only defined but also enlarged
considerably.
 Christian missionaries were also permitted to come to India and preach their religion.
 The Charter Act of 1813 marked a point of departure regarding the educational policy of East India
Company towards its Indian subjects. Under it, the Company, for the first time, accepted state
responsibility in the sphere of education. So, Statement 2 is correct.
 The Charter Act of 1833 removed All restrictions on European immigration and the acquisition of
property in India. Thus, the way was paved for the wholesale European colonisation of India. So,
Statement 3 is not correct.
 The Company’s monopoly over trade in India ended, but the Company retained the trade with China
and the trade in tea. So, Statement 4 is correct.

14. Which of the following directives were added through the 42nd Constitutional Amendment Act to Part
IV of the Indian Constitution ?
1. Equal justice and free legal aid
2. Provision for just and humane conditions of work and maternity relief
3. Protection of monuments and places, and objects of national importance
4. Protection and improvement of the environment and safeguarding of forest and wildlife
5. Organization of agriculture and animal husbandry
Select the correct answer using the code given below :
(a) 1, 3, 4 and 5 only
(b) 1 and 4 only

15
(c) 2, 3 and 5 only
(d) 1, 4 and 5 only
EXPLANATION:
The following were the amendments added to the existing list of DPSPs the 42nd Constitutional Amendment
Act to the Part IV of the Indian Constitution
 Article 39 - children are given opportunities and facilities to develop in a healthy manner and in
conditions of freedom and dignity, and that childhood and youth are protected against exploitation
and moral and material abandonment.
 Article - 39A. Equal justice and free legal aid.
The State shall secure that the operation of the legal system promotes justice on the basis of equal
opportunity and shall, in particular, provide free legal aid, by suitable legislation or schemes or in any
other way, to ensure that opportunities for securing justice are not denied to any citizen by reason of
economic or other disabilities.
 Article - 43A. Participation of workers in the management of industries.
The State shall take steps, by suitable legislation or in any other way, to secure the participation of
workers in the management of undertakings, establishments, or other organizations engaged in any
industry.
 Article - 48A. Protection and improvement of environment and safeguarding of forests and wild life.
The State shall endeavor to protect and improve the environment and safeguard the country's forests
and wildlife.
Following directive principles are embodied in original constitution:
 To make provision for just and humane conditions of work and maternity relief (Article 42).
 To protect monuments, places and objects of artistic or historic interest which are declared to be
of national importance (Article 49).
So, Option (b) is correct.

15. With respect to the recent judgements on the Representation of Peoples Act (RPA) 1951, consider the
following statements :
1. Providing false information about a candidate's educational qualification is not considered a
corrupt practice under RPA-1951.
2. Seeking votes in the name of candidates' religion, race, caste, community, or language is a corrupt
practice under RPA-1951
Which of the statements given above is/are correct ?
(a) 1 only
(b) 2 only
(c) Both 1 and 2
(d) Neither 1 nor 2
EXPLANATION:

16
The Representation of Peoples Act 1951, Section 123 of the Act, defines 'corrupt practices' to include
bribery, undue influence, false information, and the promotion or attempted promotion of "feelings of
enmity or hatred between different classes of the citizens of India on the grounds of religion, race, caste,
community, or language" by a candidate for the furtherance of his prospects in the election.
The Supreme Court observed that no one in India votes for a candidate based on their educational
qualifications, and, thus, providing false information about an electoral candidate's qualifications cannot
be considered a "corrupt practice" under Sections 123 (2) and Section 123 (4) of the Representation of
People's Act, 1951. So, Statement 1 is correct.
Section 123 (3) Representation of Peoples Act 1951 prohibits the appeals made by a candidate or his
agents to vote or refrain from voting for any person on the ground of "his" religion, race, caste, community
or language.
In Abhiram Singh v C.D. Commachen, by a 4-3 majority ruling, a seven-judge Constitution Bench held
that an election would be declared invalid if votes were sought in the name of the religion of the candidate.
So, Statement 2 is correct.

16. Consider the following statements with reference to "FINDER Technology," which was seen in the news
recently :
1. It was developed by the United States Geological Survey.
2. The technology helps in finding the aircraft that went missing.
3. It uses infrared imaging to detect minor temperature variations.
4. It helps in finding trapped individuals during post-disaster circumstances.
Which of the statements given above is/are correct ?
(a) 1 and 2 only
(b) 1, 3 and 4 only
(c) 2 and 3 only
(d) 4 only
EXPLANATION:
FINDER, which stands for Finding Individuals for Disaster and Emergency Response, is a small, suitcase-
sized device. The FINDER technology was developed by a team of NASA's Jet Propulsion Laboratory in
Pasadena, California, and the Department of Homeland Security's Science and Technology Directorate in
Washington. The latest system version was showcased at the Virginia Task Force One Training Facility in
Lorton, Virginia. It was not developed by the United States Geological Survey.
FINDER detects the small motions using algorithms similar to those that JPL uses to measure the orbits
of satellites at Jupiter and Saturn, or changes in Earth’s surface from orbiting satellites. It then displays
the detected heart and respiration rates and a reliability score. FINDER’s software can distinguish between
the heartbeats of a human and those of animals or mechanical devices.
So, Statement 1 is not correct.

17
The Finding Individuals for Disaster and Emergency Response prototype technology can locate individuals
buried under rubble in disaster scenarios. Besides natural disaster settings, the device could be used to
find people lost in a forest, trapped in a burning house or buried in the wreckage of a collapsed building.
FINDER technology is not helps to find the missing aircraft.
So, Statement 2 is not correct, and Statement 4 is correct.
FINDER is based on remote-sensing radar technology developed at NASA's Jet Propulsion Laboratory to
sense the heartbeats and breathing of humans hidden behind piles of rubble. The technology detected the
men's presence even though they were buried under about 10 feet of brick, mud, wood and other debris.
Thermal imaging is simply the process of converting infrared (IR) radiation (heat) into visible images that
depict the spatial distribution of temperature differences in a scene viewed by a thermal camera.
So, Statement 3 is not correct.

17. With reference to the President's Colour, consider the following statement :
1. Also known as Rashtrapathi Ka Nishan, it is one of the highest honors that can be awarded to any
military unit, Military training establishments and state /UT police force
2. The Indian Navy was the first Indian Armed Forces to be awarded the President's Colour in 1951.
3. It is bestowed in recognition of exceptional service rendered for the Nation only during the war.
Which of the statements given above is/are correct ?
(a) 1 only
(b) 2 only
(c) 1 and 2 only
(d) 1, 2 and 3
EXPLANATION:
The President's Colour, or the Nishan, is considered the highest honour that the Supreme Commander,
President of India, bestows on a unit for its exceptional service to the Nation. It can be awarded to any
military unit, military training establishments, and state/UT Police Forces of India. It is also known as
"Rashtrapati ka Nishaan." So, Statement 1 is correct.
Indian Navy was the first Indian Armed Force to be awarded the President's Colour by Dr. Rajendra Prasad
on May 27, 1951.
Recently INS Dronacharya, the Gunnery School of the Indian Navy, was awarded the prestigious
President's Colour on March 16, 23 by the Hon'ble President of India, Smt Droupadi Murmu.
So, Statement 2 is correct.
The President's Colour is bestowed upon a military unit in recognition of exceptional service rendered to
the Nation, both in peace and in war. So, Statement 3 is not correct.

18. Consider the following statements about taxation :


1. A proportional taxation system is meaningful when there is minimal income inequality among the
tax base.
18
2. A progressive taxation system can be beneficial to the low-income segment when there is significant
income inequality among the tax base.
3. Taxes that are applied to the value of transactions are considered proportional taxes, as high and
low-income segments pay the same amount of tax.
Which of the above statements are correct ?
(a) 1 and 2 only
(b) 1 and 3 only
(c) 2 and 3 only
(d) 1, 2 and 3
EXPLANATION:
A proportional tax is an income tax system that levies the same percentage tax to everyone regardless of
income.
For example, if the tax rate is 10%, someone who earns $50,000 would pay $5,000 in taxes, while someone
who earns $100,000 would pay $10,000 in taxes.
However, if there is significant income inequality among the tax base, a proportional taxation system may
be unfair. In this case, only a progressive taxation system may be more appropriate.
A progressive taxation system means that people with higher incomes pay a higher percentage of their
income in taxes. For example, someone who earns $50,000 might pay a tax rate of 10%, while someone
who earns $100,000 might pay a tax rate of 15%.
So it can be considered that a proportional taxation system will be meaningful when there is minimal
income inequality among the tax base. So, Statement 1 is correct.
A progressive tax involves a tax rate that increases as taxable income increases.
It imposes a lower tax rate on low-income earners and a higher tax rate on those with a higher income.
This is usually achieved by creating tax brackets that group taxpayers by income range. It is an effective
tool to reduce inequality as lower income groups are required to pay less tax and vice-versa.
Thus the progressive taxation system can be beneficial to the low-income segment when there is significant
income inequality among the tax base. Taxes that are applied on the value of transactions are considered
as progressive tax, not proportional tax. So, Statement 2 is correct, and Statement 3 is not correct.

19. Which of the following statements regarding currency management in India as per the provisions of
the RBI Act, 1934 is not correct?
(a) The Reserve Bank of India (RBI) has the sole right to issue banknotes in India.
(b) Central Bank Digital Currency has been accorded legal tender status but does not fall under
the classification of banknotes.
(c) RBI's role, in respect of coins, is limited to the distribution of coins supplied by the Government of
India.
(d) RBI, in consultation with stakeholders, estimates the denomination-wise quantity of banknotes
that are likely to be needed in a year.
EXPLANATION:

19
CBDC is the legal tender issued by a central bank in digital form and has been accorded legal tender
status, The RBI Act amendment recognizes CBDC as fiat currency, so there is no difference in the eyes of
the law or treatment between paper currency and CBDC. CBDCs enable the user to conduct domestic and
cross-border transactions, which do not require a third party or a bank.
This Concept Note explains the objectives, choices, benefits and risks of issuing a CBDC in India, referred
to as e₹ (digital Rupee). The e₹ will provide an additional option to the currently available forms of money.
It is substantially not different from banknotes, but being digital it is likely to be easier, faster and cheaper.
It also has all the transactional benefits of other forms of digital money.
The purpose behind the issue of this Concept Note is to create awareness about CBDCs in general and
the planned features of the digital Rupee, in particular. The Note also seeks to explain Reserve Bank’s
approach towards introduction of the digital Rupee. Reserve Bank’s approach is governed by two basic
considerations – to create a digital Rupee that is as close as possible to a paper currency and to manage
the process of introducing digital Rupee in a seamless manner. So, Option (b) is not correct.

20. Which of the following measures, if taken by the RBI or the government to counter excessive
depreciation of the Indian rupee, may end up being counter productive ?
(a) Raise import duty on non-essential items
(b) Encourage foreign investments in India
(c) Encourage non-resident Indians to make payments to resident Indians
(d) Discourage remittances from certain non-profit organisations
EXPLANATION:
Currency depreciation is a fall in the value of a currency in terms of its exchange rate versus other
currencies.
It refers to the decrease in the value of the domestic currency (₹) in terms of one or more foreign currencies
(like $). It makes domestic currency less valuable, and more is required to buy a currency unit.
Remittances are funds sent by individuals working or residing outside their home country to their families
or friends in their home country. These funds can be a significant source of foreign exchange and can help
support the value of the country's currency.
So, Discourage remittances from certain Non-profit Organisations, can become counterproductive in case
of tackling depreciation of money. So, Option (d) is correct.

21. If there is an excessive appreciation of the domestic currency, which of the following is a potential
outcome in an economy ?
(a) Exports become competitive and may help reduce the trade deficit
(b) Imports become cheaper and may help keep inflation in check
(c) Domestic producers find it easier to compete with imported commodities
(d) Exports and imports will be completely unaffected
EXPLANATION:

20
Currency appreciation is the rise in the domestic currency’s value compared to a foreign currency.
Currencies appreciate against each other for a variety of reasons, including the government policy, interest
rates, trade balances, and business cycles.
It enables imports to become cheaper and exports to become more expensive. It leads to higher costs of
exports, cheaper imports, lower inflation rates, etc.
The main factors contributing to currency appreciation are interest rates and inflation. In the case of low
inflation, there is an increase in interest rates, and higher rates attract more investors in the overseas
market, which will ultimately increase the value of the domestic currency.
Another main reason is investor sentiment. Suppose an investor feels that their money is safe in the
economy, i.e., political stability in the country. In that case, it will attract capital flows from overseas,
leading to an increase in the value of the domestic currency. So, Option (b) is correct.

22. Consider the following statements about India's foreign exchange reserves :
1. India's foreign exchange reserves are expressed in US dollar terms but are affected by valuation
changes of other currencies like the Euro, Pound and Yen.
2. India's foreign exchange reserves are second only to China's.
Which of the above statements is/are correct ?
(a) 1 only
(b) 2 only
(c) Both 1 and 2
(d) Neither 1 nor 2
EXPLANATION:
Foreign exchange reserve assets include foreign marketable securities, monetary gold, special drawing
rights (SDRs) and reserve position in the IMF. The main purpose of holding foreign exchange reserves is
to make international payments and hedge against exchange rate risks.
Foreign exchange reserves are maintained as a multi-currency portfolio comprising major currencies such
as the US dollar, Euro, Pound sterling, and Japanese yen, among others. Still, they are valued in terms of
US dollars.
Even though mostly expressed in dollar terms, the foreign currency assets include the effect of
appreciation or depreciation of non-US units like the euro, pound and yen held in the foreign exchange
reserves. So, Statement 1 is correct.
Forex reserves are foreign currency assets held by the central banks of countries. China has by far the
largest foreign currency reserves, with over two and a half times more than Japan's second-largest reserve
holder.
As of end-November 2022, India is the world's sixth-largest foreign exchange reserves holder.
So, Statement 2 is not correct.

23. Consider the following statements in the context of the Indian economy :
1. Gross Fixed Capital Formation is lower than Private Final Consumption Expenditure.
21
2. Per capita Gross National Product is always higher than per capita Gross Domestic Product.
Which of the statements given above is/are correct ?
(a) 1 only
(b) 2 only
(c) Both 1 and 2
(d) Neither 1 nor 2
EXPLANATION:
Gross Fixed Capital Formation (GFCF) indicates investment activity. It is a component of the Expenditure
method of calculating GDP.
Gross fixed capital formation includes spending on land improvements, machinery, and equipment
purchases; the construction of roads, railways, private residential dwellings, and commercial and
industrial buildings.
GFCF refers to the total value of fixed assets acquired by all resident producers in the economy for use in
production during a given period. In contrast, Private Final Consumption Expenditure (PFCE) refers to the
expenditure incurred by households on final goods and services.
Generally, the Gross Fixed Capital Formation is lower than Private Final Consumption Expenditure. For
example, In India, Private final consumption expenditure grew by 7 percent in Q3,2022. However, gross
fixed capital formation, an indicator of investment activity, grew by just 2 percent during the same period.
So, Statement 1 is correct.
The Per capita Gross National Product (GNP) is not always higher than the per capita Gross Domestic
Product (GDP) because GDP measures the production of goods and services made within a country's
borders by both citizens and foreign residents, whereby GNP accounts for its citizen's production both
within and outside its borders.
 Therefore, if a country has many citizens working and producing goods and services abroad, its per
capita GNP may be higher than its per capita GDP.
 Conversely, if a country has a large number of foreign workers producing goods and services within
its borders, it's per capita GDP may be higher than its per capita GNP.
So, Statement 2 is not correct.

24. Consider the following statements about Human Development Index (HDI) :
1. HDI captures only a part of what human development entails, as it does not reflect on issues like
poverty, empowerment, etc.
2. HDI can be used to raise questions on how two countries with the same level of income can have
different human development outcomes.
Which of the above statements is/are correct ?
(a) 1 only
(b) 2 only
(c) Both 1 and 2
(d) Neither 1 nor 2

22
EXPLANATION:
The HDI was created to emphasize that people and their capabilities should be the ultimate criteria for
assessing the development of a country, not economic growth alone
 The Human Development Index (HDI) is a summary measure of average achievement in key
dimensions of human development: a long and healthy life, knowledge, and decent living
standards.
 The HDI is the geometric mean of normalized indices for each of the three dimensions.
 The HDI simplifies and captures only part of what human development entails. It does not reflect
inequalities, poverty, human security, empowerment, etc.
So, Statement 1 is correct.
The Human Development Index (HDI) is a statistic developed and compiled by the United Nations since
1990 to measure various countries’ social and economic development levels.
The HDI can be used to question national policy choices, asking how two countries with the same level of
GNI per capita can end up with different human development outcomes. These contrasts can stimulate
debate about government policy priorities. So, Statement 2 is correct.

25. Consider the following statements about Marginal Standing Facility (MSF) :
1. It is a long-term borrowing facility for banks to meet their liquidity mismatch.
2. Under this facility, banks can dip into the Statutory Liquidity Ratio up to two percent of Net
Demand and Time Liabilities.
3. MSF is the ceiling of the Liquidity Adjustment Facility corridor.
Which of the above statements is/are correct ?
(a) 1 and 2 only
(b) 1 and 3 only
(c) 2 and 3 only
(d) 1, 2 and 3
EXPLANATION:

23
A marginal standing facility (MSF) is a window for banks to borrow from the Reserve Bank of India in an
emergency situation when inter-bank liquidity dries up completely. Banks borrow from the central bank
by pledging government securities at a rate higher than the repo rate under a liquidity adjustment facility
or LAF in short. Thus MSF is a short-term borrowing facility for banks to meet their liquidity mismatch.
So, Statement 1 is not correct.
The MSF would be a penal rate for banks, and the banks can borrow funds by pledging government
securities within the limits of the statutory liquidity ratio. RBI has introduced the scheme with the main
aim of reducing volatility in the overnight lending rates in the inter-bank market and enabling smooth
monetary transmission in the financial system.
According to the recent RBI, Banks will be able to dip into the Statutory Liquidity Ratio (SLR) up to two
percent of NDTL instead of three percent for overnight borrowing under the MSF with effect from January
1, 2022. So, Statement 2 is correct.
A liquidity adjustment facility (LAF) is a tool used in monetary policy, primarily by the Reserve Bank of
India (RBI), that allows banks to borrow money through repurchase agreements (repo) or to make loans
to the RBI through reverse repo agreements. This arrangement is effective in managing liquidity pressures
and assuring basic stability in the financial markets.
 The RBI introduced the LAF as a result of the Narasimhan Committee on Banking Sector Reforms
(1998).
 The RBI, in continuation of its approach towards liquidity management and normalization of the
LAF corridor, has introduced the Standing Deposit Facility or SDF at 3.75 percent and MSF at
4.25 percent. Thus, MSF is the ceiling of the Liquidity Adjustment Facility corridor.
So, Statement 3 is correct.

26. Which of the following best describes the 'Nostro account', recently seen in the news ?
(a) Refers to a bank account held in the home country by a domestic bank, denominated in the
currency of that country
(b) Refers to a bank account held in the home country by a foreign bank, denominated in the currency
of that country
(c) Refers to a bank account held in a foreign country by a domestic bank, denominated in the
currency of that country
(d) None of the above
EXPLANATION:
A Nostro account is held by one bank in another bank in its home currency. ( It refers to a bank account
held in a foreign country by a domestic bank, denominated in that country's currency). This process is
withheld in order to simplify foreign exchange and trading transactions.
 A Nostro account is a bank account that a bank holds with a foreign bank in the domestic currency
of the country where the funds are held.
 It is used to facilitate the settlement of international trade and foreign exchange transactions.

24
 The account is mainly held by banks or large corporations regularly engaging in international trade
transactions.
So, Option (c) is correct.

27. Which of the following statements about National Asset Reconstruction Company Limited (NARCL) is
not correct ?
(a) NARCL is registered with the Reserve Bank of India as an asset reconstruction company under
SARFAESI Act.
(b) It has been set up as an initiative to clean up legacy stress assets with an exposure of Rs. 500
crores or above in the Indian banking system.
(c) Security Receipts up to 100% of the value are offered by the NARCL to acquire a stressed
asset.
(d) The Security Receipts are backed by a government guarantee with a validity of five years.

EXPLANATION:
National Asset Reconstruction Company Limited (NARCL), a government entity, was incorporated with a
majority stake held by Public Sector Banks and a balance by private banks, with Canara Bank being the
sponsor bank.
 It is registered with the Reserve Bank of India as an Asset Reconstruction Company under the
Securitisation and Reconstruction of Financial Assets and Enforcement of Security Interest Act, 2002.
So, Option (a) is correct.
The Union Budget 2021 announced the formation of an ARC-AMC structure comprising two entities viz.
National Asset Reconstruction Company Limited (NARCL), and India Debt Resolution Company Limited
(IDRCL) for aggregation and resolution of Non-Performing Assets (NPAs) in the banking industry.
 NARCL has been set up with a strategic initiative to clean up the legacy stressed assets with an
exposure of Rs 500 crore and above in the Indian Banking system.
 NARCL will offer adaptable acquisition structures comprising of an optimal mix of Cash and Security
Receipts (SRs) to the Selling Banks and Financial Institutions across sectors and geographies.
 NARCL's acquisition processes will be administered by their Financial Asset Acquisition Policy framed
under the extant guidelines for Asset Reconstruction Companies.
So, Option (b) is correct.
The National Asset Reconstruction Co. Ltd (NARCL) is being established as a bad bank to take over non-
performing assets, allowing banks to focus on their core business of lending. The working structure of the
NARCL:
 The NARCL will first purchase bad loans from banks.
 It will pay 15% of the agreed price in cash, and the remaining 85% will be in the form of "Security
Receipts" (not 100%).

25
 When the assets are sold, with the help of IDRCL, the commercial banks will be paid back the rest.
 If the bad bank is unable to sell the bad loan or has to sell it at a loss, then the government guarantee
will be invoked. So, Option (c) is not correct.
NARCL's offer shall have a notable feature where a guarantee from the Government of India will back the
SRs issued by NARCL for acquiring stressed loan assets. The Guarantee shall provide NARCL a unique
strategic advantage, thereby enabling easier aggregation, which is very important for value preservation
and driving any resolution process.
Recently, The Government of India guaranteed up to Rs 30,600 crore will back Security Receipts (SRs)
issued by NARCL. The Guarantee will be valid for 5 years. The condition precedent for invocation of the
Guarantee would be resolution or liquidation.
The Guarantee shall cover the shortfall between the face value of the SR and the actual realization. Thus,
the Government's Guarantee will also enhance the liquidity of SRs as tradable.
So, Option (d) is correct.

28. Which of the following are examples of capital expenditures by businesses?


1. Purchase of machines
2. Repair work on buildings
3. Acquiring a license for new technology
4. Investing in valuables like gold
Select the correct answer using the code given below:
(a) 1, 2 and 3 only
(b) 1, 3 and 4 only
(c) 1, 2 and 4 only
(d) 2, 3 and 4 only
EXPLANATION:
Capital expenditures (CapEx) are funds used by a company to acquire, upgrade, and maintain physical
assets such as property, plants, buildings, technology, or equipment. CapEx is often used to undertake
new projects or investments by a company.
 Making capital expenditures on fixed assets can include purchasing a piece of equipment or building
a new factory. Companies make capital expenditure type of financial outlay to increase the scope of
their operations or add some future economic benefit to the operation.
 The cost of obtaining a license for new technology is a capital expenditure because it is a long-term
investment.
Capital expenditure or CapEx means cash a business spends to purchase or invest in new assets crucial
to the company's operations or creation of assets. Gold is a very useful investment during periods of
instability and high inflation. Thus, it is considered a part of capital expenditures.
Repairing work in buildings is not necessarily a part of capital expenditures. So, Option (b) is correct.

29. Which of the following statements about Primary Dealers (PDs) is correct ?
26
(a) PDs are expected to play an active role only in the primary market for G-Secs.
(b) PDs are allowed to participate only in the auctions for dated G-Secs.
(c) Only a company that is registered as an NBFC under the RBI Act, 1934, for at least one year prior
to the submission of the application will be eligible for authorization as a PD.
(d) A bank is authorized to undertake PD operations departmentally only if it doesn't partly or
wholly own any subsidiary undertaking PD business.
EXPLANATION:
The system of Primary Dealers (PDs) in the Government Securities Market was introduced by the Reserve
Bank of India in 1995 to strengthen the market infrastructure of Government Securities and put in place
an improved, efficient secondary market trading system.
This was to encourage the holding of Government Securities on a large scale and make the market more
vibrant and liquid. Primary dealers (PDs) are financial intermediaries with a mandate to take part in the
all-around development of the primary and secondary government securities market.
So, Option (a) is not correct.
PDs are expected to support the primary issues of dated securities of the Central Government and State
Government, T-Bills and Cash Management Bills (CMBs) through underwriting/bidding commitment.
So, Option (b) is not correct.
The eligibility criteria for an entity to apply to the Reserve Bank of India for undertaking the activities of a
PD are
 A subsidiary of scheduled commercial bank/s and All India Financial Institutions
 Subsidiaries/ joint ventures set up in India by entities incorporated abroad.
 The company incorporated under the Companies Act 1956 and did not fall under (a) or (b)
 Banks that do not have a partly or wholly owned subsidiary undertaking PD business and intending
to undertake PD business departmentally
So, Option (c) is not correct.
In addition to standalone PDs, banks that do not have a partly or wholly owned subsidiary may also
departmentally undertake PD business with the RBI license. Licenses are issued to banks to
departmentally undertake PD business subject to prudential and regulatory guidelines issued by RBI.
This type of departmentally undertaken PD business by the banks is known as bank PDs.
So, Option (d) is correct.

30. Labour Bureau, an attached office of the Ministry of Labour and Employment, has been compiling
Consumer Price Index for Industrial Workers. In this context, consider the following statements :
1. It is published on a monthly basis, and the index of the previous month is released on the last day
of the ongoing month.
2. The base year for the index is currently 2016.
3. The index is utilized for the regulation of wages and dearness allowances for millions of workers in
the country.
Which of the statements about the index are correct ?
27
(a) 1 and 2 only
(b) 1 and 3 only
(c) 2 and 3 only
(d) 1, 2 and 3
EXPLANATION:
Consumer Price Index Numbers for Industrial Workers (CPI-IW), which measure the rate of change in
prices of a fixed basket of goods and services consumed by the working-class population, are compiled
and maintained by the Labour Bureau, an attached office of the Ministry of Labor and Employment, since
1946.
CPI-IW has been released every month on the basis of retail prices collected from 317 markets spread over
88 industrially important centers in the country. The index is compiled for 88 centers and All-India and
is released on the last working day of the succeeding month. So, Statement 1 is correct.
The Labour and Employment Ministry revised the Consumer Price Index for Industrial Workers (CPI-IW)
base year to 2016 to reflect the changing consumption pattern, giving more weightage to spending on
health, education, recreation and other miscellaneous spending expenses while reducing the weight of
food and beverages. So, Statement 2 is correct.
Apart from measuring inflation in retail prices, the CPI-IW was The CPI (IW) indices are mainly used for
regulation of Dearness allowance and Wages of millions of Workers and Employees belonging to Central
Government, State Governments, and Public and Private sector Establishments in the country.
So, Statement 3 is correct.

31. Which of the following transactions is not a part of Non-Tax Revenue Receipts ?
(a) Profits earned from PSUs
(b) Interest-free loans received from international organizations
(c) Grants received from international organizations
(d) Interest received from loans given to other countries
EXPLANATION:
Non-Tax Revenue is the recurring income the Government earns from sources other than taxes. The non-
tax receipts of the Government are divided into three categories:
The most important receipts under Non-Tax Revenue are:
 Interest receipts (received on loans given by the Government to states, railways and others) and
dividends and profits received from public sector companies. So, Option (a) is correct
 Fees and other receipts for services rendered by the Government.
 Currency, coinage and minting- This category covers the receipts of Currency Note Press, Security
Paper Mills, Bank Note Presses and Mints, as well as the profit from the circulation of small coins.
 General services- social community services, economic services.
 Dividends and profits of RBI, Nationalised banks, public enterprises etc.
 Cash grants-in-aid from foreign countries and International Organizations. So, Option (c) is correct

28
 Interest received from loans given to other countries. So, Option (d) is correct
 Other non-tax revenue receipts: This category covers revenue from various government activities and
services such as from administrative services, public service commission, police, jails, agriculture and
allied services, industry and minerals, water and power development services, transport and
communications, supplies and disposal, public works, education, housing, information and publicity,
broadcasting, grants-in-aid and contributions etc.
Capital receipts are the huge non-recurring receipts that either create a liability or cause a reduction in
Government. These receipts, unlike revenue, are not obtained in the normal course of the Government's
financial activities. Capital receipts can be either debt-creating or non-debt-creating.
The components of the Debt receipts:
 Loans raised by the Government from the public are called market borrowings.
 Borrowing by the Government from the Reserve Bank and commercial banks, and other financial
institutions through the sale of treasury bills.
 Loans received from foreign governments and international organizations. Thus, interest-free loans are
part of capital debt receipts (not in revenue receipts). So, Option (b) is not correct.
 Other items include small savings (Post-Office Savings, Accounts, National Savings Certificates, etc.).
 Provident funds and net receipts obtained from the sale of shares in Public Sector Undertakings (PSUs)

32. Consider the following statements :


1. Capital-intensive industries will help to achieve inclusive growth in India.
2. Leapfrogging of the Indian economy from agriculture to services is one of the reasons for an increase
in unemployment in India.
Which of the statements given above is/are correct ?
(a) 1 only
(b) 2 only
(c) Both 1 and 2
(d) Neither 1 nor 2
EXPLANATION:
Capital intensive refers to a productive process that requires a high percentage of investment in fixed
assets (machines, capital, plant) to produce.
In recent years, technological development has enabled increased capital intensity in many industries.
New technology such as Artificial Intelligence, micro-computers and the internet have enabled previously
labor-intensive industries (taxi’s, delivery to become more capital intensive – e.g., with the rise of self-
driving cars.
 Increased capital intensity can cause some workers to lose their jobs because they are no longer
needed. This can lead to structural unemployment – at least in the short-term. Moreover, capital-
intensive industries may also have negative environmental and social impacts, such as pollution,
resource depletion, and displacement of local communities.

29
Thus, the Capital-intensive industries will not necessarily helps achieve inclusive growth in India So,
Statement 1 is not correct.
Leapfrogging occurs when a nation bypasses traditional stages of development to either jump directly to
the latest technologies (stage-skipping) or explore an alternative path of technological development
involving emerging technologies with new benefits and new opportunities (path-creating).
The leapfrogging of the Indian economy from agriculture to services has contributed to structural changes
in the economy, leading to significant shifts in employment patterns. While it is true that the growth of
the services sector has resulted in an increase in unemployment in some sectors, particularly in
agriculture, it is important to consider the broader trends and factors that contribute to employment
patterns in India.
Thus, Leapfrogging of Indian economy from agriculture to services resulted in increase of unemployment
in India. So, Statement 2 is correct.

33. Which of the following statements are correct with respect to the "PM VIKAS Scheme' launched recently
?
(a) Aimed at improving the skills of artisans, ensuring easy credit and also helping them in
brand promotion so that their products reach the markets quickly.
(b) Aimed at skilling the poor urban woman to provide them with employment and livelihood
opportunities
(c) Aimed at improving the digital literacy of the rural and urban poor.
(d) Aimed at improving the skills of our workforce according to the requirements of Industrial
Revolution 4.0.
EXPLANATION:
Prime Minister Vishwakarma Kaushal Samman (PM-VIKAS) scheme has been announced for traditional
artisans and craftsmen in the budget 2023-24. The scheme will enable the country's artisans to improve
the quality, scale, and reach of their products, integrating them with the Micro, Small and Medium-scale
Enterprises (MSME) value chain.
The PM VIKAS scheme aims to improve artisans' skills, ensuring easy credit and also helping them in
brand promotion so that their products reach the market quickly. It also seeks to handhold artisans and
people associated with small businesses.
Key components of the PM-VIKAS scheme include:
 Financial support
 Access to advanced skill training
 Knowledge of modern digital techniques and efficient green technologies
 Brand promotion
 Linkage with local and global markets
 Digital payments
 Social security
So, Option (a) is correct.
30
34. Consider the following statements regarding Phytorid Technology developed by CSIR :
1. The technology involves a constructed wetland exclusively designed for the treatment of
wastewater.
2. It can treat both municipal and urban waste but not agricultural and industrial wastewater.
3. It is a combination of physical, chemical and biological treatment of wastewater.
Which of the above statements is/are correct ?
(a) 1 and 3 only
(b) 2 only
(c) 2 and 3 only
(d) 1 and 2 only
EXPLANATION:
CSIR-NEERI's technology involves a constructed wetland exclusively designed for the treatment of
municipal, urban, agricultural, and industrial wastewater.
The system is based on specific plants, such as Elephant Grass (Pennisetum purpureum), Cattails (Typha
sp.), Reeds (Phragmites sp.), Cannas pp. and Yellow flag iris (Iris pseudocorus), which are normally found
in natural wetlands with filtration and treatment capability. Some ornamental and flowering plant species,
such as Golden Dhuranda, Bamboo, Nerium, Colosia, etc., can also be used for treatment and landscaping
purposes. So, Statement 1 is correct.
It uses certain specific plants that can absorb nutrients directly from wastewater but do not require soil.
These plants act as nutrient sinkers and removers. Using Phytorid Technology for the treatment of sewage,
it is possible to recover and reuse the treated water for gardening purposes
For the treatment of wastewater generated in the form of:
 Municipal/ Domestic wastewater
 Commercial establishments
 Agricultural runoffs
 Stormwater runoff
So, Statement 2 is not correct.
Phytorid Technology is a combination of physical, biological, and chemical processes. It is a technology
based on gravity that requires less energy. Phytorid Technology can be built into a series with modules /
related cells depending on land availability and the amount of wastewater to be treated in groundwater.
So, Statement 3 is correct.

35. Consider the following pairs :


Dance forms of State
India
1. Bihu - Arunachal Pradesh
2. Danda Nacha - Odisha
3. Faruwahi - Jammu and Kashmir
31
4. Purulia Chhau - West Bengal
How many pairs given above is/are correctly matched ?
(a) Only one pair
(b) Only two pairs
(c) Only three pairs
(d) All four pairs
EXPLANATION:
The Bihu dance is a folk dance from the Indian state of Assam (not in Arunachal Pradesh) related to the
festival of Bihu. This joyous dance is performed by both young men and women and is characterized by
brisk dance steps, rapid hand movement, and a rhythmic swaying of the hips in order to represent
youthful passion. Dancers wear traditionally colorful Assamese clothing.
The most important and colorful of the three Bihu festivals is the Spring festival "Bohag Bihu" or Rangali
Bihu, celebrated in the middle of April. Songs sung in Bihu are woven around themes of love and often
carry erotic overtones. People adorn traditional attires like Dhoti, Gamocha and, Chadar, Mekhala.
Bihu dances performed by young boys and girls characterized by brisk stepping, flinging and flipping of
hands and swaying of hips represent youthful passion, reproductive urge and 'Joie-de-vivre.’ So, Pair (1)
is not correct.

Danda Nacha is very popular in the Southern part of Odisha. Danda Nata of Odisha, also known as the
‘Danda Jatra.’ Danda Nacha is an unusual ritual performed by devotees in the Ganjam District of Odisha
to appease Goddess Kali. Danda Nacha means ‘Punishment Dance,’ and in the ritual, devotees inflict pain
on themselves.
At the time of Goddess Bashaktis Chaitra Festival, this Danda Nacha is celebrated in a grand manner
with much enthusiasm. Performed over a period stretching over thirteen days, Danda Nacha is a robust
show of devotion to Lord Shiva and Goddess Kali.
So, Pair (2) is correct.
Faruwahi folk dance is from the State of Uttar Pradesh, not Jammu and Kashmir. The dance form has
been in the news recently as the performance of this dance was organized by the Uttar Pradesh Global
Investors Summit 2023. So, Pair (3) is not correct.

32
Purulia is a district in West Bengal from where the famous Chhau dance originated. The word Chhau
means Chhaya (mask or shadow) according to ancient Sanskrit texts. The choreography of the dance is
taken from old rituals and dances associated with the worship of Lord Shiva. Chhau has three different
categories representing three different states. These are the Purulia Chhau of West Bengal, the Seraikella
Chhau of Jharkhand and the Mayurbhanj Chhau of Odisha.
It is said that this dance originated from the worship of Lord Shiva, where men used to put on makeup to
portray God, followed by a Tandava form of dance. The mask came much later. Its dance technique is
essentially an expression of the lives of the people and shows the intimacy between man and nature. The
performance depicts the episodes from the Mahabharata, the Ramayana, and the Puranas through dance
and music.
So, Pair (4) is correct.

36. Consider the following statements :


1. Manjusha Art is often referred to as Snake Paintings.
2. Patua art would tell Mangal kavya about gods and goddesses.
3. Pithora paintings depict animals, particularly horses.
Which of the Statements given above are correct ?
(a) 1 and 3 only

33
(b) 2 and 3 only
(c) 1 and 2 only
(d) 1, 2 and 3
EXPLANATION:
 Manjusha art is an ancient folk art of Ang Pradesh (the ancient Anga Mahajanapada). Ang Pradesh is
currently known as Bhagalpur City of Bihar. The Sanskrit word Manjusa means a ‘box.’ These boxes
were made from bamboo, Jute-Straw, and paper, inside which the devotees kept their ceremonial
materials.
 The boxes were illustrated with paintings that tell a tale. And the tale was of Bihula, who saved her
husband from the deity’s wrath and a snake bite and also of Bishahari or Mansa. The festival of
Bisahari is celebrated on the 17th and 18th of August every year. During this pooja, two things are
made. One is the “Kalash,” and the other the Manjusa.
 The Kalash is made by the Kumbhakar and Manjushas by the Malakars. Each is decorated with the
art of Manjusha, which depicts these stories and is immersed in the lake at the end of the festival.
 They worship Bisahari as they believe they will get strength from her and her protection. She promises
that their husbands will be protected from all the snakes.
 Bishaharis are represented in the same way, except they can be distinguished by what they hold in
their hands: Jaya Bishahari holds a bow and arrow with an amrit kalash in one hand and snake in
the other, Dhotila Bishahari has a rising sun in one hand and a snake in the other, Padmavathi
Bishahari has lotus in one hand, and a snake in the other, Mynah Bishahari has mynah in one hand,
and a snake in the other, and Maya/Manasa Bishahari holds snakes in both hands.
 Manjusha Art or Manjusha Kala is often referred to as Snake Paintings by foreigners as swirling snakes
in the art depict the central character Bihula’s tale of love and sacrifice.
So, Statement 1 is correct.

 Patua Painting is an Indian folk painting that originated in West Bengal. This type of art is traditionally
drawn on a piece of cloth known as a pati (or patta).

34
 The paintings are stitched onto the scroll, which is strengthened by adding fabric from old saris to the
back. Sometimes, one scroll can have a scene or panel from a longer story painted on it. It could also
have images of animals or scenes made up by the artist.
 Patuas, like other traditional painters, began by painting scrolls or patua depicting the mangal stories
of the gods and goddesses.
 For generations, scroll painters or patuas have gone from village to village telling the mangal stories of
the gods and goddesses. People found these scrolls to be a great source of entertainment.
 Traditionally, Patua painters used a brush made of bamboo and goat hair. Today, scrolls also depict
current affairs, history, and other subjects apart from traditional themes. Painters use vegetable dyes
with vegetable gum fixed on paper.
 The Patua is an artisan community found in the Indian states of West Bengal, Bihar, Jharkhand,
Odisha, and parts of Bangladesh. Some Patuas are Hindu, while others are Muslim. So, Statement 2
is correct.

 The Pithora paintings are done by the Rathwa, Bhil, Nayak and Tadi tribes of Gujarat, parts of
Rajasthan and Madhya Pradesh.
 The Pithora wall paintings are done on the main wall of the house, which divides the verandah from
the kitchen. This part of the house is considered sacred to Pithoro, the god of food grains and the
principal deity of the tribe.
 One of the most well-represented figures in Pithora wall paintings is the horse, which is used to
represent the gods, goddesses and ancestors. The depiction of seven horses in the paintings represents
the seven hills bordering their region.
 The main characters of Pithoro and Pithorani are made in white. The other deities depicted are Ramdev
and Walan, the rain god.
 The walls for Pithora wall paintings are prepared by the kumaris, or unmarried girls, using cow dung
and clay over seven days. Yet the paintings are done by men in groups of seven to eight.

35
 The characteristic white background is the result of the white clay, Pandurya. It is holy to the Rathwa
tribe and is believed to purify the background of the painting. The colours used in the Pithora wall
paintings are red, yellow, blue, green and orange.
So, Statement 3 is correct.

37. Which of the following is not a Neolithic site ?


(a) Gufkral
(b) Burzahom
(c) Koldihwa
(d) Inamgaon
EXPLANATION:
In the Kashmir valley, several Neolithic sites are near Srinagar and between Baramulla and Anantnag.
These include Burzahom, Gufkral, Hariparigom, Jayadeviudar, Olchibag, Pampur, Panzgom, Sombur,
Thajiwor, Begagund, Watzal, Gurhoma Sangri and Damodara.
During the Pleistocene era, the Kashmir valley was a gigantic lake, and the Neolithic sites are located on
the remnants of the ancient lake beds known as areas.
Mud-plastered pit dwellings have been discovered at Neolithic sites like Burzahom and Gufkral. They have
generally been interpreted as winter homes of Neolithic people. The people may have used these pit
dwellings to cope with the harsh Kashmir winter. So, Options (a) and (b) are correct.
Koldihwa is among the 40 neolithic sites identified in the Vindhyan fringes in southern Uttar Pradesh. The
Neolithic culture in this area emerged out of a well-established Mesolithic phase. Some of the Mesolithic
features, such as microlith blades and the range of heavier stone tools, continue. Still, there are also
important new features, such as cattle domestication and rice cultivation. So, Option (c) is correct.
Inamgaon is an important Malwa culture site of the Chalcolithic phase of prehistory. It is located on a
terrace of the Ghod, a tributary of the river Bhima. It is one of Maharashtra's largest, most intensively and
extensively excavated chalcolithic sites.

36
Inamgaon excavations threw light on the way of life of the Chalcolithic people of the Deccan – the way they
obtained their food and stored it, the animals they hunted and domesticated, the tools they used, beads
and pendants they used etc. So, Option (d) is not correct.

38. Consider the following statements regarding Civil Services during British rule in India :
1. Lord Wellesley founded Fort William College in Calcutta in 1800 to train civil servants.
2. Lord William Bentick was in favour of removing the humiliating distinction between Europeans
and Indians in Public Service.
3. The Charter Act of 1833 had a clause that no Indians were to be denied employment under a
company based on religion, race, color, birth, or descent.
4. The Charter Act of 1853 dissolved the company’s patronage over civil services, and; services were
now thrown open to all through competitive exams.
Which of the statements given above are correct ?
(a) 1, 2 and 3 only
(b) 1 and 3 only
(c) 1 and 4 only
(d) 1, 2, 3 and 4
EXPLANATION:
Fort William College was established by the Governor General of Bengal, Lord Wellesley in 1800. Its object
was to effect moral and intellectual development in newly recruited European civil servants.
Wellesley envisioned ruling India and British India efficiently with the help of an enlightened and trained
bureaucracy. Here, the civil servant of the company was trained in the local languages and cultures of
India. During its short history, the institution had some major achievements:
 The college staff became well known in the East and the West as interpreters of the Oriental civilization.
Their works and ideas attracted the attention of European Orientalists.
 The most outstanding students from this institution became well-renowned orientalists within the first
decade of the establishment of the college.
 The teachers and alumni of the college have been instrumental in reforming and modernizing almost
all the languages of India, including Bengali.
 It was with the encouragement and cooperation of the college that the technology of printing and
publishing vernacular books began.
The Court of Directors disapproved of this college as it negatively influenced their patronage in the
recruitment of civil servants to the company. The East India College at Haileybury in England was
established in 1806 to train civil servants of the company in Britain. So, Statement 1 is correct.
In the colonial administrative structure, the Indians were accommodated, if at all, only in subordinate
positions, known as the Uncovenanted Civil Service. After 1813, under Lord Hastings, a gradual process
of Indianization of the subordinate services began, mainly in the Judiciary. But it was Lord William
Bentinck who advocated the inclusion of Indians by removing the humiliating distinction between

37
Europeans and Indians public service and for orienting administration to local needs. So, Statement 2
is correct.
Section 87 of the Charter Act of 1833 declared that “No Indian subject of the Company in India shall by
reason only of his religion, place of birth, color or any of them, be disabled from holding any place, office
or employment under the Company.” This provision became very important as it was a bold step to remove
disqualifications.
The importance of this provision lies in the fact that it became the sheet anchor of political agitation in
India towards the end of the 19th century. Almost all the political activities in the earlier years of the
national awakening turned on this clause, which came in very handy when demands were being made for
giving equal opportunities in administration. So, Statement 3 is correct.
The Charter Act of 1833 introduced competition for the recruitment of civil servants, but it was limited
competition among the candidates nominated by the directors. It was the Charter Act of 1853 that
introduced the principle of open competition. Civil servants after the passage of this Act were to be
recruited through an examination open to all “natural born subjects of Her Majesty.” This dissolved the
company’s patronage over civil servants as the Hailey bury College was abolished in 1858, and the Civil
Service Commission recruited civil servants through an examination held annually in England.
So, Statement 4 is correct.

39. With reference to Congress Socialist Party, consider the following statements :
1. It tried to incorporate radical socio-economic measures into Indian National Congress’s (INC)
agenda by working outside the INC.
2. Congress Socialist Party didn’t participate in Quit India Movement on the same line as the
Communists.
Which of the statements given above is/are correct ?
(a) 1 only
(b) 2 only
(c) Both 1 and 2
(d) Neither 1 nor 2
EXPLANATION:
The Congress Socialist Party (CSP) was formed in Bombay in October 1934 under the leadership of
Jayaprekash Narayan, Acharya Narendra Dev and Minoo Masani. From the beginning, all the Congress
socialists agreed upon four basic propositions:
 That the primary struggle in India was the national struggle for freedom and that nationalism was a
necessary stage on the way to socialism.
 That Socialists must work inside the Indian National Congress because it was the primary body leading
the national struggle.
 That they must give the Congress and the national movement a socialist direction.

38
 That to achieve this objective, they must organize the workers and peasants in their class
organizations, wage struggles for their economic demands and make them the social base of the
national struggle.
The Congress Socialists believed in class struggle and stood for the abolition of Capitalism, Zamindari and
princely states. They wanted to incorporate radical socio-economic measures for the uplift of toiling
masses into the INC’s programme by working within the INC. So, Statement 1 is not correct.
The Communists did not join the Quit India movement; in the wake of Russia (where the communists
were in power) being attacked by Nazi Germany, the communists began to support the British War against
Germany and the ‘Imperialist War’ became the ‘People’s War.’
Unlike the Communist Party of India, which adopted the Peoples’ War line in December 1941 following
Hitler’s attack on the Soviet Union and asked its cadre to stay away from the Quit India Movement, the
Congress Socialist members played a leading role in the Quit India movement as they were more
nationalist and less ideological compared to the Communists. So, Statement 2 is not correct.

40. Consider the following statements :


1. Trade Union Act 1926 secured both civil and criminal immunity for trade unions from prosecution
for legitimate activity.
2. Trade Dispute Act of 1929 made strikes illegal in Public Utility Services.
Which of the statements given above is/are correct ?
(a) 1 only
(b) 2 only
(c) Both 1 and 2
(d) None the above
EXPLANATION:
The Trade Union Act of 1926:
 Recognized trade unions as legal associations
 Laid down conditions for registration and regulation of trade union activities
 Secured immunity, both civil and criminal, for trade unions from prosecution for legitimate activities
but put some restrictions on their political activities
So, Statement 1 is correct.
Alarmed at the increasing strength of the trade union movement under the extremist influence, the
government resorted to legislative restrictions. It passed the Public Safety Ordinance (1929) and the Trade
Disputes Act (TDA), 1929. The TDA, 1929
 Made compulsory the appointment of Courts of Inquiry and Consultation Boards for settling
industrial disputes
 Made illegal the strikes in public utility services like the post, railways, water and electricity unless
each individual worker planning to go on strike gave advance notice of one month to the
administration

39
 Forbade trade union activity of coercive or purely political nature and even sympathetic strikes.
So, Statement 2 is correct.

41. Consider the following events that occurred during the Maratha rule :
1. First attack of Surat
2. Treaty of Purander
3. Battle of Salher
4. Coronation of Shivaji
Which of the following is the correct chronological sequence of the above events ?
(a) 1-3-2-4
(b) 2-1-4-3
(c) 1-2-3-4
(d) 2-3-4-1
EXPLANATION:
Battle of Surat, also known as the Sack of Surat, was a land battle that took place on January 5, 1664,
near the city of Surat, in present-day Gujarat, India, between Maratha ruler Shivaji and Inayat Khan,
a Mughal commander. The Marathas defeated the Mughal force and ransacked the city of Surat for six
days.
The Treaty of Purandar was signed on June 11, 1665, between Jai Singh I, commander of the Mughal
Empire, and Shivaji. Shivaji Maharaj was forced to sign the agreement after Jai Singh besieged Purandar
Fort.
When Shivaji Maharaj realized that war with the Mughal Empire would only cause damage to his empire
with the possibility of his men suffering heavy losses, he chose to enter the treaty instead of leaving his
men under the Mughals.
The Battle of Salher occurred in February 1672 between the Maratha Empire and the Mughal Empire. The
fighting took place near the Salher Fort in the Nashik district. The outcome was the Maratha Empire’s
decisive victory. This war is important because it is the first time the Marathas defeated the Mughal
Dynasty.
Shivaji crowned himself and established his Swarajya (independent kingdom) on June 6 , 1674, after
recovering a large number of his forts, including Purandar, from the Mughals. He assumed the title of
“Chhatrapati” (a metaphor for “supreme king”).
Thus, the chronological order of the following events: First sack of Surat > Treaty of Purander > Battle of
Salher > Coronation of Shivaji. So, Option (c) is correct.

42. Both Buddhism and Jainism gave five main doctrines or codes of conduct. In light of this, consider
the following :
1. No violence
2. Observe continence
3. Do not lie
40
4. Do not acquire property
5. Do not use intoxicants
Which of the above doctrines are given by both Buddhism and Jainism ?
(a) 1 and 3 only
(b) 2 and 4 only
(c) 1, 3 and 5 only
(d) 1, 3, 4 and 5 only
EXPLANATION:
Jainism taught five doctrines:
 Do not commit violence
 Do not tell a lie
 Do not steal
 Do not hoard (wealth)
 Observe continence
The Buddha also laid down a code of conduct for his followers on the same line as those of the Jaina
teachers. The principle tenets are:
 Do not commit violence
 Do not covet the property of others
 Do not use intoxicants
 Do not tell a lie
 Do not indulge in sexual misconduct and adultery.
The Doctrines that were given by both Buddhism and Jainism are:
 Do not commit violence
 Do not tell a lie
So, Option (a) is correct.

43. Consider the following events in Modern Indian history :


1. Formation of All India Kisan Sabha
2. Tebhaga Movement
3. Eka movement
4. Bardoli satyagraha
Which of the following is the correct chronological sequence of the above events ?
(a) 2-3-4-1
(b) 3-2-4-1
(c) 3-4-1-2
(d) 4-2-3-1
EXPLANATION:

41
The All-India Kisan Sabha (AIKS) was founded in Lucknow in April 1936 with Swami Sahajanad Saraswati
as the president and N. G. Ranga as the general secretary. A Kisan manifesto was issued, and a periodical
was started under Indulal Yagnik.
 The AIKS and the Congress held their sessions in Faizpur in 1936 (the First Congress session in a
rural area). The Congress manifesto's agrarian agenda for the 1937 provincial elections was strongly
influenced by the AIKS agenda.
The Tebhaga movement (1946–1947) was significant peasant agitation initiated in Bengal by the All-India
Kisan Sabha of the peasant front of the Communist Party of India. Started in September 1946
 It was an intense peasant movement in the history of India. It was a fierce peasant uprising on the eve
of India's independence and the partition of Bengal.
Towards the end of 1921, peasant discontent resurfaced in some northern districts of the United
Provinces—Hardoi, Bahraich, and Sitapur. The issues involved were:
(i) high rents—50 percent higher than the recorded rates;
(ii) oppression of thikadars in charge of revenue collection; and
(iii) practice of share-rents.
The meetings of the Eka or the Unity Movement involved a symbolic religious ritual in which the assembled
peasants vowed that they would
 pay only the recorded rent but would pay it on time;
 not leave when evicted;
 refuse to do forced labor;
 give no help to criminals;
 abide by panchayat decisions.
The grassroots leadership of the Eka Movement came from Madari Pasi and other low-caste leaders and
many small zamindars. By March 1922, severe repression by authorities brought the movement to an
end.
Bardoli Satyagraha was organized by the Kunbi-Patidar land-owning peasants and untouchables,
supported by the Mehta brothers in Surat in 1928.
The Bardoli Satyagraha was a farmers' agitation and nationalist movement in India against the increased
taxation of farmers by the colonial government. It demanded a cancellation of the 22% tax hike being
levied in the Bombay Presidency. The movement began on 12 June 1928. It was eventually led by Sardar
Vallabhbhai Patel, and its success gave rise to Patel becoming one of the main leaders of the independence
movement.
Thus, the chronological order of the events is as follows: Eka movement> Bardoli satyagraha>
Formation of All India Kisan Sabha.> Tebhaga Movement
So, Option (c) is correct.

44. Consider the following statements :


Statement I: Indus Valley civilization had a social hierarchy, and the Society was multiclass.

42
Statement II: The aristocrats and rich people lived in stone buildings on an upraised citadel, while
others lived in the lower part of the city.
Choose the correct answer using the code given below :
(a) Both Statement I and Statement II are individually true, and Statement II is the correct explanation
of Statement I
(b) Both Statement I and Statement II are individually true, but Statement II is not the correct
explanation of Statement I
(c) Statement I is true, but Statement II is false
(d) Statement I is false, but Statement II is true
EXPLANATION:
Excavations at major sites of the Indus Valley Civilization, like Harappa and Mohenjo-Daro, indicate a
hierarchy in urban habitation.
 The upraised Citadels were where the ruling class lived, and the common people inhabited the lower
town.
 The middle settlement may have been meant for bureaucrats and middle-class merchants. However,
whether hierarchy in settlement corresponds to occupational divisions or socio-economic
differentiation is not clear. Hence, Statement I is correct.
The Harappan Society was multiclass. Society was multi-ethnic and Secular in outlook. Society was
materialistic in nature on the basis of the predominance of mother goddesses in the Harappan Civilization.
Sir John Marshal opined that the Harappan Society was Matriarchal in nature. Archaeological pieces of
evidence indicate that Harappan were peace-loving people and lived well-settled life. Hence, Statement I
is correct.
The upraised citadel complexes in Harappan cities were fortified with massive limestone rubble and mud
brick walls. Harappan buildings in the cities were constructed using mud bricks (not using stones).
Hence, Statement II is not correct.
So, Option (c) is correct.

45. Consider the following statements :


1. Annie Besant and Sarojini Naidu were the only women Presidents of INC before independence.
2. During the Quit India movement, Sumati Morarjee, who helped Achyut Patwardhan in his
underground activities, later became India’s leading woman industrialist.
Which of the statements given above is/are correct ?
(a) 1 only
(b) 2 only
(c) Both 1 and 2
(d) Neither 1 nor 2
EXPLANATION:
Annie Besant was the first woman president of the Indian National Congress session at Calcutta in 1917.
Sarojini Naidu became the second woman and the first Indian woman to preside over Congress after Annie
43
Besant in the Kanpur session of INC in 1925. But they were not the only women to preside over Congress
sessions before independence.
Nellie Sengupta (née Edith Ellen Gray) was an Englishwoman who fought for Indian Independence after
she relocated to India post her marriage to Jatindra Mohan Sengupta. She was elected president of the
Indian National Congress at its 48th annual session at Calcutta in 1933.
Nellie participated in the Non-Cooperation Movement of 1921. She also forcefully protested against the
district authority’s imposition of a ban on assembly, addressed mass meetings and courted arrest. She
defied the law by selling Khadi (hand-spun cloth) door to door. In 1931 she suffered four months'
imprisonment in Delhi for addressing an unlawful assembly.
When many senior leaders of the party were arrested during Salt Satyagraha, Nellie was appointed as the
President of Congress She became the third woman to be elected to the post. She was awarded the Padma
Vibhushan by the Government of India in 1973. So, Statement 1 is not correct.
Annie Besant Sarojini Naidu Nellie Sengupta

The brutal and all-out repression by the colonial government following Gandhi’s famous ‘Do or Die’ speech
at Gowalia tank succeeded within a period of six or seven weeks in bringing about a cessation of the mass
phase of the struggle. But in the meantime, underground networks were being consolidated in various
parts of the country. An all-India underground leadership with prominent members such as Achyut
Patwardhan, Aruna Asaf Ali, Ram Manohar Lohia, Sucheta Kripalani, Chootubhai Puranik, Biju Patnaik,
R.P. Goenka and later, after his escape from jail, Jayaprakash Narayan had also begun to emerge.
Those actually involved in the underground activity may have been few, but they received all manner of
support from a large variety of people. Businessmen donated generously. Sumati Morarjee, who later
became India’s leading woman industrialist, for example, helped Achyut Patwardhan to evade detection
by providing him with a different car every day borrowed from her unsuspecting wealthy friends. Others
provided hide-outs for the underground leaders and activists.
Sumati Morarjee was included in the managing agency of the company in 1923 at age 14. Sumati built
the company from humble beginnings parlaying a few vessels in the company and gradually developing it
till she assumed full charge of the company by 1946, managing over six thousand people. She was already

44
on the board of directors, and her expertise in the shipping trade, developed over many years. Due to her
amazing feat, she was also elected the president of the Indian National Steamship Owners' Association in
1956 and the next two years and again in 1965. It was under her supervision that the company rose to a
fleet of 43 shipping vessels totaling 552,000 tonnes of dead weight.
From 1979 to 1987, she was chairperson of the company until the government took over the debt-ridden
Scindia Steam Navigation. She was later appointed as the chairperson emeritus of the company till 1992.
So, Statement 2 is correct.

46. Consider the following pairs :


Authors Literary Work
1. Bhavbhuti - Maltimadhava
2. Hastimalla - Subhadra Haran
3. Kamandaka - Nitisara
4. Amogavarsha - Kavirajamargha
How many pairs given above is/are correctly matched ?
(a) Only one pair
(b) Only two pairs
(c) Only three pairs
(d) All four pairs
EXPLANATION:

45
Bhavabhuti was a dramatist who lived at Kanyakubja (modern-day Kannauj in U.P) in the early 8th
century. He has three surviving plays, namely Malatimadhava, Mahaviracharita and Uttararamacharita.
So, Pair (1) is correct.
Hastimalla was a 13th-century Kannada poet and playwright in the Hoysala empire. He wrote
Purvapurana. He wrote 8 plays, including Vikrant Kaurava and Subhadra Harana.
So, Pair (2) is correct.
Kamandaka’s Nitisara was a work written during the Gupta period (4 th century CE). It is a work on polity
addressed to the king. Kamandaka’s Nitisara advises the king to be like a florist or milkman in matters of
taxation. Just as cows have to be tended to at certain times and milked at others, and just as a florist
takes care of his plants and sprinkles water on them, besides cutting them, similarly, the king should
help his subjects with money and provisions at certain times and tax them at others. Kamandaka also
warns sternly that royal officials who become rich through ill-gotten gains should be bled like a surgeon
bleeds a swelling abscess. So, Pair (3) is correct.
Amoghavarsha, or Nripatunga of Manyakheta, was a Rashtrakuta king who is famous for building a new
capital city of Manyakheta (identified with modern Malkhed). He was a patron of literature and a scholar
himself. He wrote the Kavirajamarga, which is considered to be the earliest Kannada work on poetics.
So, Pair (4) is correct.

47. Consider the following pairs :


Commission Reason
1. Butler - To examine British India and Princely
states relationship.
2. Hartog - Education system
3. Aitchison - Public Services
4. Andrew - To draft a famine code
Frazer
How many pairs given above is/are correctly matched ?
(a) Only one pair
(b) Only two pairs
(c) Only three pairs
(d) None of the above
EXPLANATION:
Butler Committee (1927) was set up to examine the nature of the relationship between the princely states
and the government. It gave the following recommendations:
 Paramountcy must remain supreme and must fulfill its obligations, adopting and defining itself
according to the shifting necessities of time and the progressive development of states
 States should not be handed over to an Indian Government in British India, responsible to an Indian
legislature, without the consent of states

46
Thus, “paramountcy” was left undefined, and this hydra-headed creature was left to feed on usage, the
Crown’s prerogative and the princes’ implied consent. So, Pair (1) is correct.
The Hartog Committee was established by the Simon Commission in 1929 as an auxiliary committee,
chaired by Phillip Hartog, to prepare a report on education. An increase in the number of schools and
colleges had led to the deterioration of education standards. Its main recommendations were as follows:
 Emphasis should be given to primary education, but there need be no hasty expansion or compulsion
in education
 Only deserving students should go in for high school and intermediate stage, while average students
should be diverted to vocational courses after VIII standard.
 For improvements in the standards of university education, admission should be restricted.
So, Pair (2) is correct.
Aitchison Committee on Public Services was set up in 1886 by Lord Dufferin. The Committee
recommended:
 Dropping of the terms ‘covenanted’ and ‘uncovenanted’
 Classification of the civil services into Imperial, Provincial and Subordinate Civil services
 Raising the age limit to 23. So, Pair (3) is correct.
During the Viceroy-ship of Lord Curzon (1899-1905), a Police Commission was appointed under Sir
Andrew Frazer to review police administration. One of its key recommendations was the establishment of
a centralized police intelligence system, which was considered to be a major success in improving police
intelligence gathering and analysis. The Commission also recommended the separation of the executive
and judicial functions of the police and the creation of a police training school. Overall, the Frazer
Commission's recommendations had a significant impact on the Indian police system and many of its
recommendations were implemented. So, Pair (4) is not correct.

48. With reference to the Hoysala dynasty, consider the following statements :
1. Belur, situated on the banks of river Yagachi, was the capital of this kingdom during the early
stage.
2. During Alauddin Khalji’s conquest of the south, Vira Ballala III was the ruler of the Hoysala
kingdom.
3. They are contemporaries of the Bahmani Sultanate.
Which of the above statements is/are correct ?
(a) 1 and 2 only
(b) 2 only
(c) 1 and 3 only
(d) 1, 2 and 3
EXPLANATION:
The Hoysalas of Dwarasamudra emerged in the Kannadiga region and occupied prominence during the
11th-14th centuries in South India, with Belur as the center of activities. Belur was the first capital city of

47
the Hoysalas. The Chennakeshava temple complex, was at the center of the old walled town located on
the banks of the Yagachi River. The complex itself was walled in a rectangular campus with four rectilinear
streets around it for ritual circumambulation of the deity. So, Statement 1 is correct.
After the spectacular success at Warangal, in late 1310, the Delhi Sultanate ruler Alauddin Khalji sent
his general Malik Kafur on an expedition to the southernmost regions of India. In February 1311, Malik
Kafur besieged the Hoysala capital Dwarasamudra, and the defending ruler Veera Ballala III surrendered
without much resistance. Ballala agreed to pay the Delhi Sultanate an annual tribute, and surrendered a
great amount of wealth, elephants and horses. So, Statement 2 is correct.
By 1336, the Sultan had conquered the Pandyas of Madurai, the Kakatiyas of Warangal and the tiny
kingdom of Kampili. The Hoysalas were the only remaining empire who resisted the invading armies of the
Sultan. However, the ultimate signs of disintegration crept up in the time of Ballal III’s reign when
Alauddin’s forces under the leadership of Malik Kafur overran the territories in 1310-1311 CE, and Ballala
III had to shift his base to Tiruvannamalai. Veera Ballala III stationed himself at Tiruvannamalai and
offered stiff resistance to invasions from the north and the Madurai Sultanate to the south. Then, after
nearly three decades of resistance, Veera Ballala III was killed at the Battle of Kannanur in 1343, the
battle that was fought between the forces of the great Hoysala monarch Veera Ballala III and the Sultan
of Madurai, Ghiyas-ud-Din.
The Bahmani Sultanate was a Persianate Sunni Muslim empire of the Deccan in South India. The
Sultanate was founded in 1347 by Ala-ud-Din Bahman Shah. It later split into five successor states that
were collectively known as the Deccan sultanates. It was the first independent Muslim kingdom of the
Deccan, and was known for its perpetual wars with its rival Vijayanagara, which would outlast the
Sultanate. Hence the Hoysalas are not the contemporaries of the Bahmani Sultanate.
So, Statement 3 is not correct.
49. Consider the following statement about Quit India Movement :
1. Failure of August Offer of the main cause of the movement.
2. Congress demanded for an immediate end to British Rule.
3. Wavell's Plan was offered in the midst of the movement.
Which of the statements given above is/are not correct ?
(a) 1 and 2 only
(b) 1 and 3 only
(c) 2 and 3 only
(d) 1, 2 and 3
EXPLANATION:
The Failure of the Cripps Mission (1942) to solve the constitutional deadlock was one of the reasons for
the start of the Quit India Movement. The Cripps mission exposed Britain’s unchanged attitude toward
constitutional advance. It made it clear that any more silence would be tantamount to accepting the British
right to decide the fate of Indians without consulting them. Congress, on rejecting the August Offer (1940),
launched the individual satyagraha. So, Statement 1 is not correct.

48
The Quit India resolution was ratified at the Congress meeting at Gowalia Tank in Bombay on August 8 th,
1942. It resolved to:
 Demand an immediate end to British rule in India
 Declare the commitment of Free India to defend itself from all types of fascism and imperialism
 Form a provisional government of India after the British withdrawal
 Sanction a civil disobedience movement against British rule.
Gandhi was named the leader of the struggle. So, Statement 2 is correct.
After the end of the Second World War in Europe in May 1945, the Conservative government in Britain
led by Churchill was keen to reach a solution to the constitutional question in India. The viceroy, Lord
Wavell, was permitted to start negotiations with Indian leaders.
Wavell’s idea was to reconstruct the governor general’s executive council with greater Indian
representation pending the preparation of a new constitution. Wavell’s plan (1945) came after the Quit
India Movement. So, Statement 3 is not correct.

50. Which of the following statements regarding Jainism is correct ?


(a) Sallekhana is a ritual during which Jains repent for their sins and remind themselves not to repeat
them.
(b) It does not believe in the existence of soul.
(c) The Gomateswara statute at Shravanabelagola is of Lord Bahubali, the first Jain Tirthankar.
(d) The first Jain Council was convened at Pataliputra by Sthulabhadra, the leader of the
Svetambaras.
EXPLANATION:
Pratikraman is a ritual during which Jains repent for their sins during their daily life, and remind
themselves not to repeat them. While in extreme Jain austerity/ Sallekhana the body is slowly starved to
death. Since the founding of the religion, Jainism has given prominence to Sallekhana, death by ritual
fasting facing north, as exemplified in the deaths of Bhadrabahu and Chandragupta Maurya.
So, Option (a) is not correct.
The existent reality in Jainism consists of three basic categories – Sentient (that which has consciousness),
material, and neither sentient nor material. The Sentient category is represented by the jiva (Soul). Hence
Jainism believed in the existence of the soul.
According to the Jaina doctrine, the Jiva has three main qualities – consciousness, bliss and energy.
So, Option (b) is not correct.
Shravanabelagola is an important Jain pilgrimage Centre in South Karnataka. Shravanabelagola is home
to the 18 m high statue of Lord Gometeshwara, considered to be one of the world’s tallest free-standing
monolithic statues. Constructed in 981 AD by Chamundaraya, a Ganga warrior, it is carved out of a single
block of granite and looms on the top of Vindhyagiri Hill.

49
Bahubali, a much-revered figure among Jains, was the son of Rishabadeva (The First Jain Tirtankara)
and the brother of Bharata Chakravartin. He is said to have meditated motionless for a year in a standing
posture and that during this time, climbing plants grew around his legs.
So, Option (c) is not correct.
After 200 years of Mahavira’s death, a council was convened in Pataliputra, modern Patna, under the
leadership of Sthulabhadra, who stayed back in Magadha during the famine. But the Jainas who had
returned from the south boycotted it and refused to accept its decisions. From then onwards, the
southerners began to be called ‘digambaras’ and the Magadhans ‘shwetambaras.’
So, Option (d) is correct.

51. Which of the statements given below is not correct with respect to United Nations Security Council’s
Counter-Terrorism Committee :
(a) It was established by the Security Council resolution 1373, which was adopted in the wake of the
9/11 terror attacks in the US.
(b) Counter-Terrorism Committee consists of only the Permanent members of the United
Nations Security Council.
(c) Recently India hosted a special meeting to discuss terror financing through crypto-currency and
the use of drones in new-age terrorism.
(d) CTED was mandated to cooperate with the Office of the United Nations High Commissioner for
Human Rights and other human rights organizations.
EXPLANATION:
In the aftermath of the 11 September attacks against the United States in 2001, the Security Council
unanimously adopted resolution 1373 (2001), which established a dedicated Counter-Terrorism
Committee (CTC) of the Council for the first time. The CTC is assisted by an Executive Directorate (CTED),
which makes policy decisions and conducts expert assessments of the 193 United Nations Member States.
So, Option (a) is correct.
The Committee comprises all 15 Security Council members. Five permanent members: China, France,
Russian Federation, the United Kingdom, and the United States, and ten non-permanent members were
elected for two-year terms by the General Assembly.
The Committee was tasked with monitoring the implementation of resolution 1373 which requested
countries to implement several measures to enhance their legal and institutional ability to counter
terrorist activities at home and around the world. So, Option (b) is not correct.
The United Nations Security Council Counter-Terrorism Committee held a Special Meeting on Countering
the Use of New and Emerging Technologies for Terrorist Purposes in India from 28- 29 October 2022.
The Permanent Representative of India (Ruchira Kamboj) to the UN (United Nations) serves as the Chair
of the CTC for 2022.
India listed five points for the consideration of the CTC,
 Effective and sustained efforts to counter terror financing.

50
 Normative efforts of the UN need to be coordinated with other forums like the Financial Action
Task Force (FATF).
 Ensure that the Security Council’s sanctions regime is not rendered ineffective for political
reasons.
 International cooperation and concerted actions against terrorists and their sponsors, including
dismantling terrorist safe havens, etc., are critical imperatives.
 Recognize these linkages and strengthen multilateral efforts to break against terrorism’s
nexus with a transnational organized crime like arms and illicit drug trafficking.
So, Option (c) is correct.
In 2004, with the establishment of its Executive Directorate (CTED), the Counter-Terrorism Committee
began moving towards a more proactive policy on human rights. CTED was mandated to liaise with the
Office of the United Nations High Commissioner for Human Rights (OHCHR) and other human rights
organizations in matters relating to counter-terrorism, and human rights experts were appointed to its
staff.
In 2006, the Committee adopted human rights policy guidance for CTED. The Committee and CTED now
routinely take account of relevant human rights and the rule of law issues in all their activities, including
country visits, thematic meetings, technical assistance recommendations, and other interactions with
Member States. So, Option (d) is correct.

52. Consider the following statements with respect to the G20 forum :
1. All the proceedings of the G20 summit are led by the Sherpa track.
2. Turkey, Indonesia and Saudi Arabia are the only countries having membership in both G20 and
the Organization of Islamic Cooperation.
3. ‘The Voice of Global South summit’ is an initiative of India during its G20 presidency.
Select the correct answer using the code given below :
(a) 1 and 2 only
(b) 2 and 3 only
(c) 3 only
(d) 1, 2 and 3
EXPLANATION:
The Group of Twenty, or G20, is the premier forum for international cooperation on the most important
aspects of the international economic and financial agenda. It brings together the world’s major advanced
and emerging economies.
The preparatory process for the G20 Summit is conducted through the established Sherpa and Finance
tracks that prepare and follow up on the issues and commitments adopted at the Summits. The Sherpas’
Track focuses on non-economic and financial issues, such as development, anti-corruption and food
security, while addressing internal aspects, such as procedural rules of the G20 process. The Sherpas
carry out important planning, negotiation and implementation tasks continuously.

51
The Finance Track focuses on economic and financial issues. The Sherpa and Finance track relies on
expert working groups' technical and substantive work. Additionally, the thematic agenda is developed by
organizing several Ministerial Meetings, such as the Joint Meeting of Finance and Development Ministers
and the Labour, Agriculture and Tourism Ministerial meetings. Thus all the proceedings of the G20 are
led both by the Sherpa and the Finance tracks. So, Statement 1 is not correct.
The G20 comprises Argentina, Australia, Brazil, Canada, China, EU, France, Germany, India, Indonesia,
Italy, Japan, Mexico, Russia, Saudi Arabia, South Africa, South Korea, Turkey, UK and USA. The G20
Countries represent around 90% of the global GDP, 80% of global trade, and two-thirds of the world’s
population.
The Organisation of Islamic Cooperation (OIC)- was founded in 1969, with a mission to promote Islamic
solidarity and cooperation among its member states.
The OIC, headquartered in Jeddah (Saudi Arabia), is an international organization consisting of 57 member
states (with 48 countries being Muslim-majority countries, including Saudi Arabia, Indonesia, and Turkey)
Turkey, Indonesia and Saudi Arabia are the only countries having membership in both G20 and the
Organisation of Islamic Cooperation. So, Statement 2 is correct.

“The Voice of global south summit-2023’ themed as “Energy Security and Development: Roadmap to
Prosperity” is an initiative of India during its G20 presidency.
The session served as a platform to generate ideas from the developing world for achieving energy security,
which is affordable, accessible and sustainable.
India expressed willingness to support the partners in the global south by sharing experiences and best
practices and providing cutting-edge technology in the field of Biofuels, Hydrogen, and Solar, including
patented indoor solar cooking system technology to ensure energy security, transition and shared
development goals. So, Statement 3 is correct.

53. Consider the following statements with respect to National Policy on Biofuels-2018 and its Amendment
in 2022 :

52
1. A target of 20% blending of ethanol in petrol is proposed by Ethanol Supply Year (ESY) 2025-26.
2. The policy allows the use of surplus food grains for the production of Biofuels.
3. Fuels produced from lignocellulosic feedstock energy crops and non-food energy crops are
classified as 'Advanced Biofuels'.
Which of the statements given above is/are correct ?
(a) 1 and 2 only
(b) 1 and 3 only
(c) 2 only
(d) 1, 2 and 3
EXPLANATION:
India's Ministry of Petroleum and Natural Gas published its "National Policy on Biofuels" in 2018 and
further amended it in June 2022.
The policy's objective is to reduce the import of petroleum products by fostering domestic biofuel
production.
The following are the main amendments approved to the National Policy on Biofuels:
 To allow more feedstocks for the production of biofuels,
 To advance the ethanol blending target of 20% blending of ethanol in petrol to ESY 2025-26 from 2030,
 To promote the production of biofuels in the country, under the Make in India program, by units
located in Special Economic Zones (SEZ)/ Export Oriented Units (EoUs),
 To add new members to the NBCC.
 To grant permission for the export of biofuels in specific cases, and
 To delete/amend certain phrases in the Policy in line with decisions taken during the meetings of the
National Biofuel Coordination Committee.
So, Statement 1 is correct.
The National Policy on Biofuels – 2018 inter-alia allows the use of surplus food grains for production of
ethanol for blending with petrol with the approval of National Biofuel Coordination Committee (NBCC).
The NBCC in its meeting held on 2020 has allowed the production of ethanol from surplus rice available
with the Food Corporation of India (FCI) and maize respectively for blending with petrol under Ethanol
Blended Petrol (EBP) programme.
Farmers are at a risk of not getting appropriate price for their produce during the surplus production
phase. Taking this into account, the Policy allows use of surplus food grains for production of ethanol for
blending with petrol with the approval of National Biofuel Coordination Committee.
The National Bio-fuels Policy, 2018 seeks to widen the range of feedstock for ethanol production from the
sugar molasses to other waste, such as rural-urban garbage and cellulosic and lingo-cellulosic biomass,
in line with the “waste-to-wealth” concept.
The permissible feedstock includes sorghum, sugar beet, cassava, decaying potatoes, damaged grain
including maize, wheat, rice, and most importantly, crop residues such as wheat and rice stubble.
So, Statement 2 is correct.

53
Advanced Biofuels, also called Second-generation biofuels, are liquid fuels generally derived from
lignocellulosic biomass, nonfood crop feedstocks, agricultural & forest residues, and industrial wastes
that yield a lifecycle reduction in greenhouse gas emissions of at least 50% compared with fossil fuels.
So, Statement 3 is correct.

54. Which of the following are the business areas of New Space India Limited (NSIL), the commercial arm
of the Indian Space Research Organisation (ISRO) ?
1. Production of Polar Satellite Launch Vehicle (PSLV) and Small Satellite Launch Vehicle (SSLV)
2. Transponder leasing
3. Building Communication and Earth Observation Satellites
4. Transfer of technology developed by ISRO
5. Marketing of products emanating out of ISRO activities
Select the correct answer using the code given below :
(a) 2, 3 and 4 only
(b) 1, 2, 3 and 4 only
(c) 2, 4 and 5 only
(d) 1, 2, 3, 4 and 5
EXPLANATION:
New Space India Limited (NSIL), incorporated on 6 March 2019 (under the Companies Act, 2013), is a
wholly owned Government of India company under the administrative control of the Department of Space
(DOS). NSIL is the commercial arm of the Indian Space Research Organization (ISRO) with the primary
responsibility of enabling Indian industries to take up high-technology space-related activities. It is also
responsible for promoting and commercializing the products and services emanating from the Indian space
program.
To satisfy the needs of its customers, NSIL draws upon the proven heritage of the Indian Space Program
and ISRO’s vast experience in diverse branches of Space Technology.
The major business areas of NSIL include:
 Production of Polar Satellite Launch Vehicle (PSLV) and Small Satellite Launch Vehicle (SSLV) through
industry;
 Production and marketing of space-based services, including launch services and space-based
applications like transponder leasing, remote sensing and mission support services;
 Building of Satellites (both Communication and Earth Observation) as per user requirements.
 Transfer of technology developed by ISRO centers/ units and constituent institutions of Dept. of Space;
 Marketing spin-off technologies and products/ services emanating out of ISRO activities.
 Consultancy services
So, Option (d) is correct.

55. Which of the following is/are a part of the Innate Immune system in Human beings ?
1. Skin
54
2. Gastric juice
3. White blood cells
4. Cytokine signaling protein
Select the correct answer using the code given below :
(a) 1 and 2 only
(b) 3 and 4 only
(c) 3 only
(d) 1, 2, 3 and 4
EXPLANATION:
The innate immune system is the body's first line of defense against germs entering the body. It responds
similarly to all germs and foreign substances, which is why it is sometimes called the "nonspecific" immune
system.
Innate immunity involves barriers that keep harmful materials from entering the body. These barriers
form the first line of defense in the immune response.
The innate immune system consists of
 The protection offered by the skin and mucous membranes
 The protection offered by the immune system cells (defense cells) and proteins
Examples of innate immunity include:
 Cough reflex
 Enzymes in tears and skin oils
 Mucus, which traps bacteria and small particles
 Skin
 Stomach acid
Innate immunity also comes in a protein chemical form called innate humoral immunity. Examples
include the body's complement system and substances called interferon and interleukin-1 (which cause
fever).
If an antigen gets past these barriers, it is attacked and destroyed by other parts of the immune system.
Many of the cells in the innate immune system (such as dendritic cells, macrophages, mast cells,
neutrophils, basophils and eosinophils) produce cytokines or interact with other cells directly to activate
the adaptive immune system. So, Option (d) is correct.

55
56. Which of the following are hyperglycaemic hormones which will increase the blood sugar level ?
1. Insulin
2. Glucagon
3. Growth Hormone
4. Cortisol
5. Adrenaline
6. Noradrenaline
Select the correct answer using the code given below :
(a) 1, 2, 3 and 4
(b) 2, 3 and 5 only
(c) 2, 3, 4, 5 and 6 only
(d) 1, 2, 3, 4, 5 and 6 only
EXPLANATION:
Glycemia is the presence of sugar (glucose) in the blood. Hyperglycaemia indicates excess glucose in the
blood. Hypoglycaemia refers to abnormally low presence of glucose in the blood.
The hyperglycaemic hormone is secreted by the alpha cells of the islets of Langerhans, which stimulates
the release of glucose into the blood. Epinephrine (adrenaline), cortisol, growth hormone, and glucagon
are hormones that help maintain blood sugar levels. They are called “stress” or “gluco-counter-regulatory”
hormones and raise blood sugar levels.

56
Norepinephrine, also known as noradrenaline, has effects similar to epinephrine, including increased
blood sugar levels, heart rate, and contractility. It can also narrow blood vessels, leading to increased
blood pressure.
Insulin is a Hypoglycemic hormone (not a Hyperglycaemic hormone) that reduces blood sugar by helping
the cells use glucose. So, Option (c) is correct.

57. “The spacecraft will explore Titan’s diverse environments and take advantage of its dense nitrogen-
based atmosphere to fly like a drone. The instruments will investigate the moon’s atmospheric and
surface properties, subsurface ocean, liquid reservoirs, and areas where water and complex organic
materials key to life once existed together for possibly tens of thousands of years.” The experiment in
question refers to :
(a) Dragonfly Mission
(b) SOFIA
(c) Europa Clipper
(d) Ingenuity
EXPLANATION:
The Dragonfly mission will be launched in 2026 and land in 2034 on Saturn's largest moon, Titan, the
only celestial body besides our planet known to have liquid rivers, lakes and seas on its surface, though
these contain hydrocarbons like methane and ethane, not water. NASA’s Dragonfly spacecraft taking
advantage of Titan, Saturn’s exotic moon’s dense atmosphere and low gravity, will explore dozens of
locations across the icy world, sampling and measuring the compositions of Titan's organic surface
materials to characterize the habitability of Titan’s environment and investigate the progression of
prebiotic chemistry. So, Option (a) is correct.
SOFIA is a telescope on a modified Boeing 747SP aircraft jointly operated by NASA and DLR- Germany.
It is the world's largest airborne observatory and can make observations that are impossible for ground-
based telescopes.
SOFIA's mission includes studying the formation of solar systems, identifying complex molecules in space,
and observing planets, comets, and asteroids in the Solar System. So, Option (b) is not correct.
NASA's Europa Clipper will conduct detailed reconnaissance of Jupiter's moon Europa and investigate
whether the icy moon could have conditions suitable for life. The mission will place a spacecraft in orbit
around Jupiter. So, Option (c) is not correct
NASA's Ingenuity Mars Helicopter has completed 50 flights since first taking to the skies above the Red
Planet on April 19, 2021, far exceeding its originally planned technology demonstration of up to five flights.
So, Option (c) is not correct
58. India has recently installed a 4-meter International Liquid Mirror Telescope (ILMT). Which of the
following are the benefits of a Liquid Mirror Telescope compared to a reflective telescope ?
1. Large aperture and wide field of view
2. Low weight
3. Easy maintenance
57
4. Consistent observation/uptime
5. Less impact on the environment
6. Variable tilt/orientation
7. Complete Electromagnetic spectral range
Select the correct answer using the code given below :
(a) 1, 2, 3, 4, 5 and 6 only
(b) 1, 2, 3 and 5 only
(c) 3, 4, 6 and 7 only
(d) 1, 2, 3, 4, 5, 6 and 7
EXPLANATION:
The International Liquid-Mirror Telescope (ILMT) is the first liquid-mirror telescope set up exclusively for
astronomical observations at the Devasthal Observatory campus owned by Aryabhatta Research Institute
of Observational Sciences (ARIES), Nainital in Uttarakhand.
 Devasthal Observatory now hosts two four-metre class telescopes – the ILMT and the Devasthal Optical
Telescope (DOT). Both are the largest aperture telescopes available in the country.
 One advantage of a liquid mirror telescope is that it can be constructed at a much lower cost than a
traditional telescope, as the mirror surface can be easily and inexpensively produced. Additionally,
LMTs are lightweight, making them easier to transport and install.
 The scientists spun a pool of mercury which is a reflective liquid, so that the surface curved into a
parabolic shape which is ideal for focusing light. A thin transparent film of mylar protects the mercury
from wind. The reflected light passes through a sophisticated multi-lens optical corrector that
produces sharp images over a wide field of view.
 A conventional telescope is steered to point towards the celestial source of interest in the sky for
observations. Liquid mirror telescopes cannot be tilted and hence cannot track the way
conventional telescopes do. The tracking is done artificially by using a technique called Time Delay
Integration (TDI).
 Operational wavelength of Liquid-Mirror Telescope is 4000 to 11000 Å spectral range only while the
electromagnetic spectrum has the range of 0.01 Å to 109 Å. Thus it cannot cover the Complete
Electromagnetic spectral range.

58
 Another difference between the two is their operational time. While conventional telescopes observe
specific stellar sources for fixed hours as per the study requirement and time allotted by the respective
telescope time allotment committee, ILMT will capture the sky’s images on all nights — between two
successive twilights. Hence the liquid mirror telescope doesn’t have Consistent observation/uptime.
 A liquid mirror telescope can have less impact on the environment in comparison to traditional
telescopes because it can be constructed with fewer materials, requires less energy to operate, and can
be dismantled and removed with minimal environmental disruption.
So, Option (b) is correct.

59. Consider the following statements :


1. Cultured meats are grown from animal cells, while plant-based meats do not use animal cells.
2. In terms of cellular structure, cultured meat is the same as conventional meat.
3. Cultivated meat uses much less land and water than livestock and produces no methane
emissions.
Which of the statements given above are correct ?
(a) 1 and 2 only
(b) 1 and 3 only
(c) 2 and 3 only
(d) 1, 2 and 3
EXPLANATION:
Plant-based meats are created by several methods, but the most popular technique is known as the “high-
moisture extrusion”. The Plant-proteins are put inside a barrel before going through thermal and
mechanical stresses – a process that involves a variety of heating, cooling, and shearing techniques – to
produce a product that is essentially plant proteins with a meat-like texture. By altering the texture
process slightly, the characteristics of each plant-based meat can also be changed.

59
On the other hand, the “cultured meat”, lab-grown meat takes on a different approach to creating
‘alternative’ meats. Lab-grown meat uses muscle samples and stem cells from animals, then cultivates
(i.e., grows) this small sample into a large amount of meat. So, Statement 1 is correct.

Cultivated meat, also known as cultured meat, is genuine animal meat (including seafood and organ
meats) that is produced by cultivating animal cells directly. This production method eliminates the need
to raise and farm animals for food.
Cultivated meat is made of the same cell types that can be arranged in the same or similar structure as
animal tissues, thus replicating the sensory and nutritional profiles of conventional meat. So, Statement
2 is correct.
Cultivated meat has several benefits over traditional livestock meat production.
 Firstly, it does not harm animals as cells are extracted once from a donor animal and can produce
cultured meat indefinitely. This eliminates the need for harmful procedures used in traditional farming
and slaughter.
 Secondly, it frees up land as livestock meat production requires 70% of global arable land used for
growing livestock feed. Cultivation of animal cells requires smaller quantities of agricultural resources
like land, reducing land usage for meat production by 99%.
 Thirdly, it uses considerably fewer water resources compared to conventional meat production,
reducing water usage by 82%-96%.
 Fourthly, the Cattle systems are associated with the production of all three GHGs i.e. carbon dioxide
(CO2), methane (CH4), and nitrous oxide (N2O), including significant emissions of CH4, while cultured
meat emissions are almost entirely CO2 from energy generation.
 Finally, there is a growing demand for meat worldwide and cultivated meat provides another meat
protein option that can alleviate pressure from conventional supply chains while using considerably
fewer natural resources to produce meat.
So, Statement 3 is correct.

60
60. The term ‘Bortle Scale’ seen in the news recently is in the context of :
(a) Measurement of the night sky's brightness
(b) Scale to sort marine fossils based on size
(c) Measurement of the magnetic field generated by the core of an astronomical object
(d) Scale to measure the Supervised learning of an Artificial Intelligence (AI) model
EXPLANATION:
The Bortle scale is a way of measuring the quality (brightness) of the night sky for a particular location.
The Bortle Dark-Sky Scale is a nine-level numeric scale that measures the night sky's and stars' brightness
(naked-eye and stellar limiting magnitude) of a particular location. It quantifies the observability of
celestial objects (significant naturally occurring physical entities, associations or structures which current
science has demonstrated to exist in outer space) and the interference caused by light pollution and
skyglow (wide scale illumination of the sky or parts of the sky at night). The most common cause of skyglow
is man-made lights that give off light pollution. So, Option (a) is correct.

61. Consider the following pairs :


COVID-19 vaccines Approach
1. Covishield - Viral Vector Vaccine
2. COVAXIN - Inactivated vaccine
3. iNOVACC - Live-attenuated vaccine
4. Comirnaty - Plasmid DNA vaccine
How many pairs given above is/are correctly matched ?
(a) Only one pair
(b) Only two pairs
(c) Only three pairs
(d) All four pairs
EXPLANATION:
The Oxford–AstraZeneca COVID‑19 vaccine, sold under the brand name Covishield is a viral vector vaccine
which is used for the prevention of COVID-19.

61
A Viral Vector Vaccine uses a modified version of a virus (a vector) that cannot cause disease but serves
as a platform to produce coronavirus proteins to generate an immune response by delivering genetic
instructions to the body's cells.
Covishield uses a replication-deficient chimpanzee viral vector based on a weakened version of a common
cold virus (adenovirus) that causes infections in chimpanzees and contains the genetic material of the
SARS-CoV-2 virus spike protein. After vaccination, the surface spike protein is produced, priming the
immune system to attack the SARS-CoV-2 virus if it later infects the body. So, Pair 1 is correct.

Inactivated vaccines use a form of the virus that has been inactivated or weakened so it doesn’t cause
disease but still generates an immune response.
COVAXIN, is India's indigenous (inactivated) COVID-19 vaccine developed by Bharat Biotech is in
collaboration with the Indian Council of Medical Research (ICMR) - National Institute of Virology (NIV).
So, Pair 2 is correct.
The nasal Covid-19 vaccine, iNCOVACC, is an Indian vaccine developed to protect against the Covid-19
virus.
Unlike the traditional Covid-19 vaccines, administered through injection, the iNCOVACC is administered
through the nose. The vaccine uses a live attenuated virus modified to be less virulent. The vaccine
contains a small amount of the Covid-19 virus, which helps the body recognize and fight the virus when
exposed.
The vaccine, a homologous live-attenuated LSD vaccine, has been developed by researchers at the National
Centre for Veterinary Type Culture and ICAR-National Research Centre on Equines (ICAR-NRCE), Hisar,
Haryana, in collaboration with ICAR-Indian Veterinary Research Institute (IVRI), Izatnagar, Uttar
Pradesh. So, Pair 3 is correct.
Messenger RNA vaccines—also called mRNA vaccines make proteins to trigger an immune response.
mRNA vaccines have several benefits compared to other types of vaccines, including shorter
manufacturing times and, because they do not contain a live virus, no risk of causing disease in the person
getting vaccinated.

62
COMIRNATY (COVID-19 Vaccine) is a mRNA vaccine (not an Plasmid NDA vaccine) indicated for active
immunization to prevent coronavirus disease 2019 (COVID-19) caused by severe acute respiratory
syndrome coronavirus 2 (SARS-CoV-2) in individuals 12 years of age and older. So, Pair 4 is not correct.

62. Which of the following events can happen when two neutron stars spiral and collide with each other?
1. Ripples of Gravitation waves will be created
2. Blackholes can be formed
3. Ejection of heavy metals such as gold and platinum
4. Gamma-ray burst
Select the correct answer using the code given below :
(a) 1, 2 and 4 only
(b) 1, 3 and 4 only
(c) 1 only
(d) 1, 2, 3 and 4
EXPLANATION:
When the two neutron stars meet, their merger leads to the formation of either a more massive neutron
star, or a black hole (depending on whether the mass of the remnant exceeds the Tolman–Oppenheimer–
Volkoff limit).
The collision creates the instance of a single source emitting ripples in space-time, known as gravitational
waves, as well as light, which is released in the form of a two-second gamma ray burst.
The collision also creates heavy elements such as gold, platinum and lead, scattering them across the
universe in a kilonova – similar to a supernova – after the initial fireball.
The LIGO Livingston Observatory picked up what appeared to be gravitational ripples from a collision of
two neutron stars. LIGO Livingston is part of a gravitational-wave network that includes LIGO (the Laser
Interferometer Gravitational-wave Observatory), funded by the National Science Foundation (NSF), and
the European Virgo detector. Now, a new study confirms that this event was indeed likely the result of a
merger of two neutron stars. This would be only the second time this type of event has ever been observed
in gravitational waves. So, Option (d) is correct.

63. Consider the following statements :


1. India has more Thorium reserves than Uranium reserves.
2. Thorium is not a fissile material.
3. Uranium reserves in India cannot be extracted on a commercial scale due to low energy density
compared to the imported Uranium.
4. Prototype Fast Breeder Reactor (PFBR) built by India will use Thorium-232 as fuel.
Which of the statements given above are correct ?
(a) 1 and 4 only
(b) 3 and 4 only
(c) 1, 2 and 3 only
63
(d) 1 and 2 only
EXPLANATION:
India has 10.70 million tonnes of Monazite which contains 9,63,000 tonnes of Thorium Oxide (ThO2).
India's thorium deposits, estimated at 360,000 tonnes, far outweigh its natural uranium deposits at
70,000 tonnes.
The country's thorium reserves make up 25 per cent of the global reserves. It can easily be used as a fuel
to cut down on the import of Uranium from different countries. So, Statement 1 is correct.
Thorium-232, the only naturally occurring isotope of thorium, is a fissionable material but not a fissile
one, meaning that it needs high-energy neutrons to undergo fission — the splitting of atomic nuclei which
releases energy that is used for electricity generation. So, Statement 2 is correct.
A group called the Rare Metal Survey Unit (later renamed as Atomic Minerals Directorate for Exploration
and Research) was formed by Govt. of India to locate good uranium deposits in the country.
The pioneering work of the geologists brought to light many uranium occurrences in the Jaduguda of
Singhbhum Thrust belt (the first uranium deposit discovered in 1951 in the eastern part of the country).
Soon, it became evident that this belt holds the potential for commercial uranium mining operations.
This discovery of uranium at Jaduguda in this belt paved the way for intensive exploration work, and soon
a few more deposits were brought to light in this area. Some of these deposits, like Bhatin, Narwapahar
and Turamdih, are well-known uranium mines in the country.
Apart from discoveries in the Singhbhum Thrust Belt, several uranium occurrences have also been found
in the Cuddapah basin of Andhra Pradesh. These include Lambapur-Peddagattu, Chitral, Kuppunuru,
Tumallapalle, and Rachakuntapalle, which have significantly contributed to the uranium reserve base of
India.
Also, other deposits like Bagjata, Banduhurang and Mohuldih are being taken up for commercial mining
operations. So, Statement 3 is not correct.

64
The Prototype Fast Breeder Reactor (PFBR) is a 500 MWe fast breeder nuclear reactor presently being
constructed at the Madras Atomic Power Station in Kalpakkam, India.
The Indira Gandhi Centre for Atomic Research (IGCAR) is responsible for the design of this reactor.
The Kalpakkam PFBR is using uranium-238 not thorium, to breed new fissile material, in a sodium-
cooled fast reactor design. However, thorium is more difficult to use than uranium as a fuel because it
requires breeding, and global uranium prices remain low enough that breeding is unnecessary. So,
Statement 4 is not correct.

64. Consider the following statements :


1. Seismic P-waves are transverse in nature, while Seismic S-waves are longitudinal in nature.
2. Sound is a mechanical wave, while the light is a non-mechanical wave.
Which of the statements given above is/are correct ?
(a) 1 only
(b) 2 only
(c) Both 1 and 2
(d) Neither 1 nor 2
EXPLANATION:
P-waves vibrate parallel to the direction of the wave. This exerts pressure on the material in the direction
of the propagation. As a result, it creates density differences in the material leading to stretching and
squeezing of the material.

65
The direction of vibrations of S-waves is perpendicular to the wave direction in the vertical plane Secondary
waves (S-waves) are transverse and shear waves in nature, while the Primary waves (P-waves) are
longitudinal and compressional waves by nature. So, Statement 1 is not correct.

Mechanical Waves are waves which propagate through a material medium (solid, liquid, or gas) at a wave
speed which depends on the elastic and inertial properties of that medium.
The most familiar type of waves such as waves on a string, water waves, sound waves, seismic waves, etc.
is the so-called mechanical waves. These waves require a medium for propagation, they cannot propagate
through vacuum. They involve oscillations of constituent particles and depend on the elastic properties of
the medium.
Not all waves require a medium for their propagation, For example, light waves can travel through vacuum.
The light emitted by stars, which are hundreds of light years away, reaches us through inter-stellar space,
which is practically a vacuum. Thus Light waves are Non-Mechanical waves.
Electromagnetic waves do not necessarily require a medium - they can travel through vacuum. Light, radio
waves, X-rays, are all electromagnetic waves. In vacuum, all electromagnetic waves have the same speed.
So, Statement 2 is correct.

65. Consider the following statements regarding the characteristics of Blockchain technology :
1. Decentralized database
2. Immutability
3. Approved reversal of fraudulent transactions
4. Unanimous
5. Anonymity of users
6. Energy saving
Select the correct option using the codes given below :
(a) 1, 2, 3, 4, 5 and 6 only
(b) 1, 2, 4 and 5 only
(c) 1, 3, 5 and 6 only
(d) 1, 2, 3, 4, 5 and 6
EXPLANATION:

66
Blockchain technology is an advanced database mechanism that allows transparent information sharing
within a business network. A block chain database stores data in blocks that are linked together in a
chain.
Key features of Blockchain Technology
 Block chain technology has a key feature called "immutability," which means that once a transaction
block is added to the digital ledger, it cannot be edited, deleted, or updated.
 Decentralized structure, which means that there is no governing authority and users have direct
control over their assets using their private key.
 Unanimous - Every participant in a block chain network agrees that each of the records is valid
 Anonymous - The identity of the participants within a network can remain either anonymous or
pseudonymous
 Hashing is Irreversible, quite complex, and it’s impossible to alter or reverse it. No one can take a
public key and come up with a private key. Also, a single change in the input could lead to a completely
different ID, so small changes aren’t a luxury in the system.
 Block chains are energy intensive, the Bitcoin block chain alone currently uses 204,5 TWh of electricity
per year, comparable to the power consumption of entire Thailand.
So, Option (b) is correct.

66. 'Paasikivi line' recently seen in the news is related to :


(a) It is a proposed conventional boundary between Earth's atmosphere and outer space.
(b) It is a strategy used by Finland to maintain a peaceful coexistence with the Soviet Union.
(c) It is a dark or bright line in an otherwise continuous and uniform spectrum.
(d) It is a resolved long-standing disputed line between Assam and Meghalaya.
EXPLANATION:
After the Second World War, Paasikivi's line emerged and began to morph into what would eventually
become Finland's foreign policy strategy.
Named after Juho Kusti Paasikivi, the president of Finland between 1946 and 1956, Paasikivi's line was
based on the idea of peaceful coexistence with the Soviet Union, with neutrality being its cornerstone. It
helped Finland navigate complex international relations after the turbulent time of the Second World War.
The Paasikivi's line aimed to maintain Finland's territorial integrity and sovereignty and avoid conflicts
between the Soviet Union and the West.
The policy remained in place until the end of the Cold War. It continues to influence Finnish foreign policy
today, with Finland remaining outside military alliances and maintaining neutrality and non-alignment.
So, Option (b) is correct.

67
67. With reference to Shanghai Cooperation Organization (SCO), consider the following statements :
1. It is an eight-member multilateral organization, with India being a dialogue partner.
2. Ujjain has been designated as the first cultural capital of the SCO.
3. SCO is a cooperative organization based on non-alignment and does not target any third party.
4. Recently SCO has become an observer in the United Nations General Assembly.
Which of the statements given above is/are correct ?
(a) 1 and 3 only
(b) 2 and 3 only
(c) 3 only
(d) 4 only
EXPLANATION:
The Shanghai Cooperation Organization (SCO) is an intergovernmental organization founded in Shanghai
on 15 June 2001. The SCO currently comprises
 Eight Member States (China, India, Kazakhstan, Kyrgyzstan, Russia, Pakistan, Tajikistan and
Uzbekistan),
 Four Observer States interested in acceding to full membership (Afghanistan, Belarus, Iran, and
Mongolia)
 Six “Dialogue Partners” (Armenia, Azerbaijan, Cambodia, Nepal, Sri Lanka and Turkey).
Thus, India is a member of SCO and not a dialogue partner.
So, Statement 1 is not correct.
Varanasi has been nominated as the first SCO Tourism and Cultural Capital during 2022-2023 at the
22nd Meeting of the Shanghai Cooperation Organization (SCO) Council of Heads of State in Samarkand,
Uzbekistan.

68
It will promote tourism and cultural and humanitarian exchanges between India and the SCO member
Countries. It also underlines India’s ancient civilizational links with Member States of SCO, especially the
Central Asian Republics.
The title ‘Cultural and Tourism Capital’ will rotate among the member states. Under this initiative, each
year, a city of the cultural heritage of a member country that will take over the rotating Presidency of the
organization will get the title to highlight its prominence.
So, Statement 2 is not correct.
The success of the organization was built upon several factors, including China-Russia cooperation, efforts
by the “Shanghai Five,” relations among member states, and external pressure. China and Russia,
working together, provided a strong impetus.
They are twin engines with leadership and demonstration roles in SCO development. The principles of
non-alignment, non-confrontation and non-targeting of any third party, coming from historical experience
and lessons learned in China-Soviet relations, ensured normal, orderly relations, guiding the organization
to a model of state-to-state relations in partnership rather than alliance. So, Statement 3 is correct.
The SCO has been an observer in the UN General Assembly since 2005 (not recently). In April 2010, the
UN and SCO Secretariats signed a Joint Declaration on Cooperation.
SCO Secretariat has also established partnerships with the UN Educational, Scientific and Cultural
Organization (UNESCO), the World Tourism Organization (UNWTO), and the International Organization
for Migration (IOM), in addition to its ongoing cooperation with the UN Office on Drugs and Crime
(UNODC), United Nations Economic and Social Commission for Asia and the Pacific (ESCAP) and the UN
Office on Counter-Terrorism (UNOCT). So, Statement 4 is not correct.

68. Consider the following statements with respect to Glasgow leaders' Declaration on Forests and Land
Use (GDFLU) :
1. It was launched at the United Nations Climate Change Conference (COP26).
2. Its objective is to collectively halt and reverse forest loss and land degradation by 2050.
3. India is a party to the declaration on forests and land use.
Which of the statements given above is/are correct ?
(a) 1 only
(b) 2 only
(c) 3 only
(d) 1 and 3 only
EXPLANATION:
The Glasgow Leaders' Declaration on Forests and Land Use (GDFLU) was launched at the November 2021
United Nations Climate Change Conference (COP26).
A total of 143 nations signed the declaration, accounting for over 90% of the world's forests. This effort
aimed to 'halt and reverse forest loss and land degradation by 2030 while delivering sustainable

69
development and promoting inclusive rural transformation' (UN Climate Change Conference, UK 2021).
So, Statement 1 is correct.
Its main goal is to halt and reverse forest loss and land degradation by 2030 while delivering sustainable
development and promoting inclusive rural transformation' (UN Climate Change Conference, UK 2021).
So, Statement 2 is not correct.
India did not sign the Glasgow Leaders' Declaration on Forests and Land Use, an ambitious declaration
initiated by the United Kingdom to "halt deforestation" and land degradation by 2030, as it objected to
"trade" being interlinked with climate change and forest issues in the agreement. The declaration has over
105 signatories, including the UK, the US, Russia, and China. Argentina, Mexico, Saudi Arabia, and South
Africa are the only G20 countries that did not sign the declaration. So, Statement 3 is not correct.

69. Consider the following pairs :


Protected area State
1. Chitwan National Park : Jharkhand
2. Tal Chhapar Sanctuary : Rajasthan
3. Kadavur Slender Loris Sanctuary : Tamil Nadu
How many pairs given above is/are correctly matched ?
(a) 1 and 2 only
(b) 2 and 3 only
(c) 1 and 3 only
(d) 1, 2 and 3
EXPLANATION:
The Chitwan National Park (CNP) Nepal's first National Park, located in the Southern Central Terai of
Nepal was established in 1973.
Chitwan National Park is a world heritage property, and it also contains a Ramsar Site – Beeshazari Tal
in its buffer zone. The CNP has a history of over 3 decades in park management and rich experience in
resolving conflicts between the park and the people.
According to the recent study, Mugger crocodiles posed a risk of anthropogenic threats in Nepal’s Chitwan
National Park, says study. Anthropogenic threats like illegal fishing and sand mining threaten the mugger
crocodiles (Crocodylus plaustris) of the Rapti river flowing along the Chitwan National Park (CNP) in south-
central Nepal, contiguous to the Valmiki Tiger Reserve in Bihar.
Researchers found that there were 46 muggers in the stretch of the Rapti that they surveyed. The animals
were most likely to be found on moderate or moderately steep banks of the river. A win-win conservation
situation between vulnerable muggers and the local community is necessary for the viable and long-term
conservation of muggers in the Rapti river. So, Pair 1 is not correct.
Tal Chhapar Sanctuary is known as the home of blackbuck (about 4,000 blackbucks) and a variety of
birds, which is located 85 km from Churu in Shekhawati District, in Rajasthan.

70
The Great India Desert flanks the sanctuary, Thar and boasts a unique ecosystem and is an important
birdwatching destination in India. It has tall grasses and is dotted with numerous small ponds.
The Tal Chhapar sanctuary gets protection from a plan to reduce its size through a suo motu public
interest litigation by the Rajasthan High Court. Before independence, Tal Chappar was a Private Hunting
Reserve of the Maharaja of Bikaner. Forest - Open grassland with spread all over Acacia and Prosopis
trees. It looks like a typical Savannah.
 Fauna - It hosts several wild animals, raptors, and resident and migratory birds. Large colonies of the
only herbivorous lizard, the spiny-tailed lizard, exist here as the prey base for raptors. A special type
of grass, called ‘Mothiya’ or pearl, is found here.
 Threats - Hyper-aridity, grazing pressure, the invasive weed Prosopis juliflora and salt mines.
 Conservation - A major project for the conservation of raptors in the sanctuary is taken up by World
Wildlife Fund for Nature (WWF).
So, Pair 2 is correct.
Tamil Nadu government has notified the country’s first slender loris sanctuary in Kadavur (Kadavur
Reserve Forest). Located in the Karur and Dindigul districts of Tamil Nadu, it covers 11,806 hectares of
the Kadavur Reserve Forest.
Slender lorises, which are small nocturnal mammals, are arboreal in nature as they spend most of their
life on trees. The species acts as a biological predator of pests in agricultural crops and benefits farmers.
Listed as an endangered species by the International Union for Conservation of Nature (IUCN), Slender
Loris has a wide range of ecological roles to play in the terrestrial ecosystem.
So, Pair 3 is correct.

70. With reference to the impact of climate change on agriculture, consider the following statements :
1. Climate change will likely reduce the yield of wheat, maize and chickpea.
2. Increase in atmospheric carbon dioxide has a fertilization effect on crops with C 3 photosynthetic
pathway and thus promotes their growth and productivity.
3. Under elevated CO2 concentration, crop residues have a higher Carbon-Nitrogen ratio, which may
reduce their rate of decomposition and nutrient supply in the soil.
Which of the statements given above are correct ?
(a) 1 and 2 only
(b) 2 and 3 only
(c) 1 and 3 only
(d) 1, 2 and 3
EXPLANATION:
Climate change is perceptible through a rise in all India's mean temperature and increased frequency of
extreme rainfall events in the last three decades. This causes fluctuation in the production of major crops
in different years.
The impact of climate change on Indian agriculture was studied under National Innovations in Climate
Resilient Agriculture (NICRA). Rainfed rice yields in India are projected to reduce marginally (<2.5%) in
71
2050 and 2080, and irrigated rice yields by 7% in 2050 and 10% in 2080 scenarios. Further, wheat yield
is projected to reduce by 6-25% in 2100 and maize yields by 18-23%. Future climates will likely benefit
chickpea with the increase in productivity (23-54%).
So, Statement 1 is not correct.
An increase in atmospheric carbon dioxide has a fertilization effect on crops with C3 photosynthetic
pathway and thus promotes their growth and productivity.
An increase in temperature can reduce crop duration, increase crop respiration rates, alter the
photosynthesis process, and affect the survival and distribution of pest populations.
Despite this, the yields of major cereals crops, especially like wheat, are likely to be reduced due to a
decrease in crop growth duration, increased respiration, and reduction in rainfall/irrigation water supplies
due to a rise in atmospheric temperature. So, Statement 2 is correct.
Climate change can affect agriculture through its direct and indirect effects on crops, soils, livestock and
pests in the following ways:
 Reduced quantity and quality of organic matter content, which is already quite low in Indian soil.
 Under elevated CO2 concentration, crop residues have higher Carbon: Nitrogen ratio, which may
reduce their rate of decomposition and nutrient supply.
 An increase in soil temperature will increase N mineralization, but its availability may decrease due
to increased gaseous losses through processes such as volatilization and denitrification.
 Changes in rainfall volume, frequency, and wind intensity may alter the severity, frequency, and
extent of soil erosion.
 A rise in sea level may lead to salt-water ingression in the coastal lands turning them less suitable for
conventional agriculture. So, Statement 3 is correct.

71. Which of the following are the pollutants from the Aluminium industry ?
1. Fluoride emissions
2. Carbon monoxide
3. Polycyclic aromatic hydrocarbons (PAHs)
4. Sulphur dioxide
5. Benzopyrene
Select the correct answer using the code given below :
(a) 1, 2 and 3 only
(b) 1, 3 and 4 only
(c) 1, 3, 4, and 5 only
(d) 1, 2, 3, 4 and 5
EXPLANATION:
Aluminium is the most widely used nonferrous metal. Metal production has increased by nearly four times
in the last three decades. Global bottled water giants are ramping up trials of easily recyclable aluminium
cans to replace plastic that pollutes the world's seas.

72
Aluminium cans might mean less ocean waste, but they come with their own eco-price: each can pump
about twice as much carbon into the atmosphere as each plastic bottle.
The most important pollutants emitted from the primary aluminium electrolysis process are:
 Red mud, the residue of the bauxite digestion treatment
 Fluoride emissions resulting from the vaporization of some of the constituents of the electrolytic bath
recent trends in alumina and aluminium production technology.
 Spent pot lining--the waste cathode material
 Particulate matter
 Carbon monoxide (CO)
 Polycyclic aromatic hydrocarbons (PAH's)
 Sulphur dioxide
 Benzopyrene
 Mercury
 Polyfluorinated hydrocarbons and fluorides are also produced during the electrolysis process.
 Dust is emitted mainly during the electrolysis stage in the primary production of aluminium.
So, Option (d) is correct.

72. With reference to recent Battery Waste Management Rules, 2022, consider the following statements:
1. The rules cover all types of batteries, viz. Electric Vehicle batteries, portable batteries, automotive
batteries and industrial batteries.
2. It mandates the use of labeling to reflect the carbon footprint of the battery.
3. Extended Producer Responsibility is introduced for all battery producers and importers.
4. It is based upon the concept of ‘Outcome oriented’, where the targets are measurable in terms of
the collection and recycling of the waste within a compliance timeframe.
Which of the statements given above are correct ?
(a) 2, 3 and 4 only
(b) 1, 3 and 4 only
(c) 1, 2 and 4 only
(d) 1, 2, 3 and 4
EXPLANATION:
Ministry of Environment, Forest and Climate Change, Government of India published the Battery Waste
Management Rules, 2022, to ensure environmentally sound management of waste batteries.
Notifying these rules is a transformative step towards implementing the announcement made by Prime
Minister Shri Narendra Modi in his address to the Nation on Independence Day to promote the Circular
Economy in full earnest.
New rules will replace Batteries (Management and Handling) Rules, 2001. The rules cover all types of
batteries, viz. Electric Vehicle batteries, portable batteries, automotive batteries, and industrial batteries.
So, Statement 1 is correct.
Labelling requirements of Battery Waste Management Rules, 2022:
73
 Producers shall ensure that all Battery or Battery packs are appropriately marked with requisite
labelling requirements as per standards prescribed by the Bureau of Indian Standards.
 All requisite labels and symbols shall be printed visibly, legibly and indelibly.
 No person shall place on the market any Battery or Battery pack unless it is marked with the
―crossed-out wheeled bin symbol.
 The size of the Battery or Battery pack is such that the crossed-out wheeled bin symbol would be
smaller than 0.5 cm x 0.5 cm, the Battery or Battery pack need not be marked, but a crossed-out
wheeled bin symbol measuring at least 1 cm x 1 cm shall be printed on the packaging.
 No person shall place a Battery or a button cell on the market containing mercury, cadmium or lead
unless it is marked with the respective chemical symbol Hg, Cd or Pb.
The Battery Waste Management Rules, 2022, does not mandate the use of labeling for reflecting the carbon
footprint of the battery. This would have made their environmental impact more transparent and reflective.
So, Statement 2 is not correct.
The Rules function based on the concept of Extended Producer Responsibility (EPR), where the producers
(including importers) of batteries are responsible for the collection and recycling/refurbishment of waste
batteries and the use of recovered materials from wastes into new batteries.
EPR mandates that all waste batteries be collected and sent for recycling/refurbishment, and its prohibits
disposal in landfills and incineration.
 To meet the EPR obligations, producers may engage themselves or authorize any other entity to collect,
recycle, or refurbish waste batteries.
 The rules will enable the setting up of a mechanism and centralized online portal for the exchange of
EPR certificates between producers and recyclers/refurbishers to fulfil the obligations of producers.
 The rules promote the setting up of new industries and entrepreneurship in the collection and
recycling/refurbishment of waste batteries.
 Penalty in the form of the ‘Polluter pay principle’ would be levied upon the person who would not
abide by the concept of EPR.
 The fund derived from environmental compensation will be used for the purpose of recycling
uncollected and non-recycled waste batteries.
So, Statement 3 is correct.

74
The remarkable feature of the new rules is that it is based upon the concept of ‘Outcome oriented’. Here
targets are measurable in terms of the collection and recycling of the waste within a compliance
timeframe. So, Statement 4 is correct.

73. Consider the following statements with reference to Methane :


1. Oceans, volcanoes, and wildfires contribute the largest percentage of global methane emissions
from natural sources.
2. Atmospheric reaction is the largest natural sink of Methane.
3. Fossil Fuel use and agriculture contribute the largest methane emission from human activities.
Which of the statements given above is/are correct ?
(a) 3 only
(b) 1 and 3 only
(c) 2 and 3 only
(d) 1, 2 and 3
EXPLANATION:
Methane (CH4) is emitted by natural sources such as wetlands, as well as human activities such as leakage
from natural gas systems and raising livestock.
Natural processes in soil and chemical reactions in the atmosphere help remove CH4 from the atmosphere.
Source of Methane emission from Natural sources:
 Wetlands are the largest source, emitting CH4 from bacteria that decompose organic materials in the
absence of oxygen.
 Smaller sources include termites, oceans, sediments, volcanoes, and wildfires.
 Wetlands provide a habitat favorable to methanogens, contributing to about 80% of the global
methane emissions from natural sources.
So, Statement 1 is not correct.
The atmosphere, and more precisely the troposphere, is the largest sink for methane. Methane in the
troposphere reacts with hydroxyl (OH) radicals, forming mainly water and carbon dioxide.
In total, this reaction accounts for about 500 million tonnes of methane each year. An indirect effect of
atmospheric methane oxidation is that it can magnify the effects of other pollutants. Increased methane
in the atmosphere means fewer OH radicals and less oxidizing power in the atmosphere. Soils act as a
major sink for atmospheric methane through the methanotrophic bacteria that reside within them.
So, Statement 2 is correct.
Globally, 50 – 65% of total CH4 emissions come from human activities. More than 50% of the global
methane emissions from human activities in 3 sectors:
1. Agriculture (40%)
2. Fossil Fuels (35%)
3. Waste (20%).

75
The Agriculture sector is the primary source of CH4 emissions. Domestic livestock such as cattle, buffalo,
sheep, goats, and camels produce large amounts of CH4 in their normal digestive process (enteric
fermentation). Also, when animals’ manure is stored or managed in lagoons or holding tanks, CH4 is
produced. Because humans raise these animals for food, the emissions are considered human-related.
Paddy rice cultivation accounts for another 8% of human-linked emissions.
Methane is the primary component of natural gas. Some amount of CH4 is emitted into the atmosphere
during the production, processing, storage, transmission, and distribution of crude oil & natural gas.
Waste from Homes and Businesses: Methane is generated in landfills as waste decomposes and from the
treatment of wastewater.
So, Statement 3 is correct.

74. With reference to Climate Change Performance Index 2023, consider the following statements:
1. Climate Change Performance Index is published annually by UNEP.
2. India’s rank is the best amongst the G20 countries in recently released
3. India earns a high rating in the GHG Emissions and Renewable Energy categories in Climate
Change Performance Index 2023.
4. As per the report, India is "on track" to meet its 2030 emissions targets.
Which of the statements given above are correct ?
(a) 1 and 3 only
(b) 1 and 2 only
(c) 2 and 4 only
(d) 1, 2, 3 and 4
EXPLANATION:
The Climate Change Performance Index has been published annually since 2005. This ranking tracks the
efforts made by different countries to deal with climate change. Key Points of Climate Change Performance
Index Report: This report has been prepared by Germanwatch, New Climate Institute and Climate Action
Network.
It is an independent monitoring tool for tracking the climate protection performance of 59 countries and
the European Union. These countries collectively account for more than 92% of global Greenhouse Gas
(GHG) Emissions.
Report leaves the first three places empty as "no country performed well enough in all index categories to
achieve an overall very high rating". It puts Denmark in fourth place, followed by Sweden and Chile.
The CCPI looks at four categories, with 14 indicators: GHG Emissions (40% of the overall score), Renewable
Energy (20%), Energy Use (20%), and Climate Policy (20%). So, Statement 1 is not correct.
India has been ranked amongst the top 5 countries in the World, and the best among the G20 countries,
based on its Climate Change performance. India jumped 2 spots higher and is now ranked 8th as per
Climate Change Performance Index (CCPI, 2023), which shows Denmark, Sweden, Chile and Morocco as
the only four small countries that were ranked above India as 4th, 5th, 6th and 7th, respectively.

76
The ranking given by CCPI places India as the only G-20 country in the top 10 ranks. India will now be
assuming the G-20 Presidency, and it will be an opportune time to show the World about its climate
mitigation policies, such as the deployment of renewable sources of energy and other energy transition
programmes.

So, Statement 2 is correct.


India earned a high rating in the GHG Emissions and Energy Use categories, while a medium for Climate
Policy and Renewable Energy.
The aggressive policies of India towards rapid deployment of renewables and robust framework for energy
efficiency programs have shown considerable impact. As per the CCPI report, India is on track to meet its
2030 emissions targets (compatible with a well-below-2°C scenario).
So, Statement 3 is not correct; Statement 4 is correct.

77
75. With reference to Oysters, consider the following statements :
1. Oysters feed by filtering algae, nutrients and suspended matter from the water.
2. Oyster reefs can serve as a barrier to storms and tides
3. Oysters are found mostly in the pelagic zone of the ocean.
Which of the statements given above is/are correct ?
(a) 1 only
(b) 2 only
(c) 1 and 2 only
(d) 1, 2 and 3
EXPLANATION:
Oysters live in salty or brackish coastal waters, clustering on older shells, rocks, piers, or any hard,
submerged surface. They fuse together as they grow, forming rock-like reefs that provide habitat for other
marine animals and plants.
Oysters are also excellent water filters – improving water quality by filtering algae, nutrients and
suspended matter from the seawater. When growing as part of a shellfish reef ecosystem, oysters also
provide a habitat for a huge range of other animals, such as octopuses, crabs, fish, sea squirts, sea snails
and sponges.
So, Statement 1 is correct.
Oyster reefs can serve as barriers to storms and tides, preventing erosion and protecting productive
estuary waters. Ongoing threats to oysters mean that, unless deliberate efforts are made to protect and
restore them, we are in danger of losing the benefits oyster reefs provide.
 An oyster reef is an ecological community made up of densely packed oysters (bivalve molluscs). It
provides a habitat for toadfish, skilletfish and blue crabs.
 Oysters suck water through gills, and suspended tiny organisms and pollutants like nitrogen are
trapped in the gills' fine lining.
 They eat the organisms and expel the rest as solid pellets of waste, which decompose and bubble up
as nitrogen.
 A healthy oyster can filter up to five litres of water per hour, thus, cleansing it of algae and pollutants.
 Oyster reefs are mostly found with mangroves along the coast. Hence they also defend shorelines and
reduce erosion. So, Statement 2 is correct.
The benthic zone is the ecological zone located at the bottom of any marine or freshwater body, such as a
river, ocean, lake or pond. The benthic zone includes the sediment surface. Benthic zones are found all
across the world, wherever there is an appreciable water body. The benthos lives on the ocean floor.
Starfish, oysters, clams, sea cucumbers, brittle stars and anemones are all benthos. Most benthos feed
on food as it floats by or scavenges for food on the ocean floor.
The pelagic zone is part of the open sea or ocean comprising the water column, i.e., all of the sea other
than that near the coast or the sea floor. Examples of species include forage fish such as anchovies,

78
sardines, shad, and menhaden and the predatory fish that feed on them. Oceanic pelagic fish typically
inhabit waters below the continental shelf. Examples include larger fish such as swordfish, tuna,
mackerel, and even sharks. So, Statement 3 is not correct.

76. Which of the following are considered as Benthos ?


1. sea anemones
2. sponges
3. corals
4. sea stars
5. sea urchins
6. worms
7. bivalves
8. crabs
Select the correct answer using the code given below :
(a) 1, 2, 4, 6 and 7

79
(b) 1, 2, 4, 5, and 6
(c) 1, 2, 3, 4, 5, 6 and 7
(d) 1, 2, 3, 4, 5, 6, 7 and 8
EXPLANATION:
The benthos comprises all the organisms that live at the bottom of a body of standing or running water.
The layer that the benthos occupy is called the benthic zone, which is the lowest layer of a lake, sea,
stream, or river.
This zone ranges from the shallow depths where water meets land to extreme depths that humans have
not yet been able to explore. Benthos, therefore, vary greatly and can either be observed crawling,
burrowing, swimming near the bottom, or staying attached to the substrate.
Most of these animals lack a backbone and are called invertebrates. Typical benthic invertebrates include
sea anemones, sponges, corals, sea stars, sea urchins, worms, bivalves and crabs.
Many benthos in shallow waters also relies on dead organic matter as their source of nutrition, breaking
it down and recycling it. This makes them very important for nutrient cycling and returning nutrients to
the environment in usable forms. Certain benthos is also useful to scientists in a way that can indicate
the health and quality of the water. For example, a decline in the number of caddisflies, which are very
sensitive to pollutants, can indicate an increase in pollution and waste in the water.
So, Option (d) is correct.

77. Consider the following statements with reference to the 19th COP 2022 to CITES :
1. India proposed to include the freshwater Red-crowned Roofed turtle from Appendix II to Appendix
I of CITES.
2. India voted against the proposal seeking to re-open the ivory trade.
3. Export of Shisham-based products is banned in India as decided in this conference.
Which of the statements given above is/are correct ?

80
(a) 1 only
(b) 2 only
(c) 1 and 3 only
(d) 1, 2 and 3
EXPLANATION:
The Convention on International Trade in Endangered Species of Wild Fauna and Flora (CITES) is also
known as the Washington Convention. The COP19 to the CITES is also known as the World Wildlife
Conference. The 19th Conference of the Parties (COP19) to CITES is being held in Panama in 2022.
 In the CITES COP19, countries have proposals to amend the appendices for wild animals to Appendix
I, including red-crowned roofed turtles and African elephants to Appendix I.
 It also proposes to include requiem and hammerhead sharks in Appendix II.
 India has proposed better protection for the red-crowned roofed turtle (Batagur kachuga) under
CITES.
The Red-crowned roofed turtle is native to India, Bangladesh and Nepal. Historically, the species was
widespread in the Ganga River, both in India and Bangladesh. It also occurs in the Brahmaputra basin.
Currently, in India, the National Chambal River Gharial Sanctuary is the only area with a substantial
species population, but even this Protected Area and habitat are under threat. So, Statement 1 is correct.

India abstained to vote against a proposal to re-open the international trade in ivory at the ongoing
conference of the Convention on International Trade in Endangered Species of Wild Fauna and Flora
(CITES).
The proposal to allow a regular form of controlled trade in ivory from Namibia, Botswana, South Africa
and Zimbabwe was defeated at the 19th meeting of the CITES (CoP19) in Panama City.
India’s abstention — a break from its past stand — was in tune with what Namibia had asked for when it
agreed to transfer cheetahs this summer.
So, Statement 2 is not correct.
Shisham (Dalbergia sissoo) is included in Appendix II of the convention, requiring the following CITES
regulations for the species trade. Currently, every consignment of weight above 10 kg requires a CITES
permit. Due to this restriction, exports of furniture and handicrafts made of Dalbergia sissoo from India
have been continuously falling which affected the livelihoods of around 50,000 artisans who work
depending on the species.
On India’s initiative, a proposal to clarify the quantity of Shisham (Dalbergia sissoo) items, such as
furniture and artefacts for export, was considered in the current meeting. It was agreed upon that any

81
number of Dalbergia sissoo timber-based items could be exported as a single consignment in a shipment
without CITES permit if the weight of each individual item of the consignment is less than 10 kg. This was
a great relief for the Indian artisans and furniture industry. Thus, India gets CITES rules eased for export
of Shisham items. So, Statement 3 is not correct.

78. Consider the following statements :


1. Biomining is the scientific process of excavation, treatment, segregation, and gainful utilization of
aged municipal solid waste lying in dumpsites, typically referred to as legacy waste.
2. Bio oxidation, Bioleaching, Dump leaching, and Agitated Leaching are various methods of
Biomining.
Which of the statements given above is/are correct ?
(a) 1 only
(b) 2 only
(c) Both 1 and 2
(d) Neither 1 nor 2
EXPLANATION:
According to the Central Pollution Control Board (CPCB) guidelines, "Biomining is the scientific process of
excavation, treatment, segregation and gainful utilisation of aged municipal solid waste lying in dumpsites
typically referred to as legacy waste." Simply put, it is a process of treating garbage or waste with bio-
organisms or natural elements like air and sunlight. Over time, the biodegradable component of the waste
gets decomposed through the natural process, and the remaining part i.e., the non-biodegradable
material, is then dealt with separately. It extracts metals of economic interest from rock ores or mine
waste.
 Biomining is a method of extracting minerals and metals from their parent ores using naturally
occurring biological processes. The practice requires none of the environmentally damaging processes
found in conventional refinement methods and instead relies entirely on the natural interaction of
biological organisms.
So, Statement 1 is correct.
Biomining comprises 4 steps: excavation of legacy waste, stabilizing the waste using bioremediation,
segregation of excavated waste, and sustainable management and its safe disposal.
The methods of biomining include Bioleaching, Bio-oxidation, Dump leaching, and agitated leaching.
 Bioleaching is a process in mining and biohydrometallurgy (natural processes of interactions between
microbes and minerals) that extracts valuable metals from low-grade ore with the help of
microorganisms.
 Bio-oxidation is a natural biological treatment process (green process) used for the biological control
of air pollution. Combusting any fuel generates oxides of nitrogen (NOx), particulate matter, sulphur
dioxide (SO2), and carbon monoxide (CO).
 Heap leaching: freshly mined material is moved directly into heaps that are then bioleached.

82
 Dump leaching: low-value ore or waste rock is placed in a sealed pit and then bioleached to remove
more of the valuable metals from the waste pile.
 Agitated Leaching: crushed rocks are placed into a large vat that is shaken to distribute the microbes
and material evenly and speed up the bioleaching process.
So, Statement 2 is correct.

79. Which of the following are the characteristics of Ecotone ?


1. It can be natural or man-made.
2. Organisms that occur primarily in this zone are known as ‘Edge species’.
3. It can have a distinct species that might not be found in other bordering ecosystems.
Select the correct answer using the code given below :
(a) 1 and 2 only
(b) 2 and 3 only
(c) 1 and 3 only
(d) 1, 2 and 3
EXPLANATION:
Ecotone is a zone of junction between two or more diverse ecosystems. E.g., the mangrove forests represent
an ecotone between marine and terrestrial ecosystems. Other examples are – grassland, estuary and river
bank. It might be small or large, local (the area between a field and a forest), or regional (the area between
a field and a forest) (the transition between forest and grassland ecosystems). Mountain ranges can also
create ecotones due to the changes in the climatic conditions on the slopes.
Characteristics of Ecotone:
 It may be very narrow or quite wide.
 It has conditions intermediate to the adjacent ecosystems. Hence it is a zone of tension.
 It is linear as it shows a progressive increase in species composition of one in the coming community
and a simultaneous decrease in species of the other outgoing adjoining community.
 A well-developed ecotones contain some organisms which are entirely different from that of the
adjoining communities.
 Ecotones can be natural or man-made. For example, the ecotone between an agricultural field and a
forest is a man-made one. So, Statement 1 is correct.
Sometimes the number of species and the population density of some species is much greater in this zone
than in either community. This is called the edge effect.
The organisms which occur primarily or most abundantly in this zone are known as edge species.
So, Statement 2 is correct.
A well-developed ecotone contain some organisms which are entirely different from that of the adjoining
communities.

83
In ecotones, certain other species that are specially adapted for living in this zone will also thrive. Such
species are often not found in the adjacent ecosystems. The variation produced in such areas contributes
to increased biodiversity. So, Statement 3 is correct.

80. Which one of the following statements is not correct ?


(a) Mahogany is a timber species of Tropical Rainforest
(b) Chaparral vegetation is found in the Taiga climate
(c) Lichens are predominantly found in the Tundra region
(d) Sal is a representative species of Moist Deciduous Forest
EXPLANATION:

84
Chaparral, vegetation is composed of broad-leaved evergreen shrubs, bushes, and small trees, usually
less than 2.5 m (about 8 feet) tall; together, they often form dense thickets. Chaparral is found in regions
with a climate similar to that of the Mediterranean area, characterized by hot, dry summers and mild, wet
winters. Nearly all of the rainfall occurs in the winter and spring rainy seasons.
Some chaparral ecosystems (the ones with the low scrubby brush) are very similar to the desert scrub
biome, but chaparral specifically occurs in coastal regions and has much more varied vegetation, whereas
desert scrub is a transitional zone between deserts and grasslands.
 The climate of the chaparral biome is very hot and dry.
 Winters there tend to be very mild, with temperatures of about 50 °F. The winter climate is also known
as the Mediterranean climate, which while not very rainy, is typically mild and moist.
 Most of the precipitation occurring in the Biome occurs during the winter months. Summers, which
tend to be hot and dry, run about 104 °F but range from 30° to 120° F.
 Summers are unique in the Biome because while it can get very hot, temperatures can quickly dip to
nearly freezing. So, Option (b) is not correct.

81. With reference to Ecosystem services, consider the following statements :


1. Millennium Ecosystem Assessment divides ecosystem services into two categories as Provisioning
and Supporting services.
2. Out of the total cost of various ecosystem services, soil formation accounts for about 50 percent.
3. Nutrient cycling and Primary production are examples of ‘Supporting Services’.
Which of the statements given above is/are correct ?
(a) 1 only
(b) 2 and 3 only
(c) 1 and 3 only
(d) 3 only
EXPLANATION:
The Millennium Ecosystem Assessment, published in 2005, divided ecosystem services into four
categories:
 Provisioning services, or the supply of goods of direct benefit to people, often with a clear monetary
value, such as timber from forests, medicinal plants, and fish from the oceans, rivers and lakes.
 Regulating services are the range of functions carried out by ecosystems that are often of great value
but generally not given a monetary value in conventional markets. They include regulation of climate
through the storing of carbon and control of local rainfall, the removal of pollutants by filtering the air
and water, and protection from disasters such as landslides and coastal storms.
 Cultural services do not provide direct material benefits but contribute to society's wider needs and
desires and, therefore, to people’s willingness to pay for conservation. They include the spiritual value
attached to particular ecosystems, such as sacred groves, and the aesthetic beauty of landscapes or
coastal formations that attract tourists.
 Supporting services is not directly beneficial to people but essential to the functioning of ecosystems

85
and, therefore, indirectly responsible for all other services. Examples are the formation of soils and the
processes of plant growth. Biodiversity is not regarded as an ecosystem service itself but rather as a
prerequisite underpinning each of them. The precise link between diversity and the capacity of an
ecosystem to provide services is a complex one and an area in which science is still developing.
So, Statement 1 is not correct.
Robert Constanza and his colleagues have very recently tried to put price tags on nature’s life-support
services. Researchers have put an average price tag of US $ 33 trillion a year on these fundamental
ecosystem services, which are largely taken for granted because they are free. This is near twice the value
of the global gross national product (GNP) which is (US $ 18 trillion).
Out of the total cost of various ecosystem services, soil formation accounts for about 50 percent, and
contributions of other services, like recreation and nutrient cycling, are less than 10 percent each. The
cost of climate regulation and habitat for wildlife is about 6 percent each.
So, Statement 2 is correct.
Ecosystem services do the following: Moderate weather extremes and their impacts (ex., drought, floods,
etc.), Mitigate climate change, Absorb and store CO2, Protect water channels and shores from erosion,
Regulate disease-carrying organisms, Provide ingredients for pharmaceuticals, and biochemical and
industrial products, Are a source of energy and biomass fuels, Decompose waste and detoxify pollution,
Generate maintain and renew soil fertility (nutrient cycling), Pollinate crops and plants, and disperse
seeds, Control agricultural pests and diseases, Produce food (such as crops, wild foods and spices,
seafood), Produce wood and fibre, Produce oxygen, purify air and water; Give cultural, intellectual, artistic
and spiritual inspiration; Allow recreation (ex. ecotourism); Hold answers to scientific questions; Hold the
cures to diseases. So, Statement 3 is correct.

82. Which of the following statements is/are correct about the Prevention of Cruelty to Animals Act 1960?

86
1. The Animal Welfare Board of India, the first of its kind to be established by any Government in the
world, was set up under this Act in 1962 and governed by the Ministry of Fisheries and Animal
Husbandry.
2. The Act permits the welfare board to receive grants from the Government but can also receive
contributions or donations from any other person.
3. The Act prohibits experimentation on animals.
4. The Act has restrictions on the exhibition and training of performing animals.
Select the correct answer using the code given below :
(a) 2 only
(b) 3 only
(c) 1, 2 and 4 only
(d) 1, 2, 3 and 4
EXPLANATION:
The Animal Welfare Board of India is a statutory advisory body on Animal Welfare Laws and promotes
animal welfare in the country. The Animal Welfare Board of India, the first of its kind to be established by
any government in the world, was set up in 1962 in accordance with Section 4 of the Prevention of Cruelty
to Animals Acts 1960. It functions under the aegis of the Ministry of Fisheries, Animal Husbandry, and
Dairy (MoFAH&D). So, Statement 1 is correct.
The Act permits the welfare board to receive the funds, which shall consist of grants made to it from time
to time by the government and contributions, donations, subscriptions, bequests, gifts and the like made
to it by any local authority or any other person. So, Statement 2 is correct.
The laws related to animal experimentation come under Chapter IV, Sections 14 to 20, of the PCA Act. It
prohibits any person from inflicting, causing, or, if it is the owner, permitting unnecessary pain or suffering
inflicted on any animal. The Act provides for punishment for causing unnecessary cruelty and suffering
to animals. So, Statement 3 is not correct.
Section 22 of The Prevention of Cruelty to Animals Act, 1960 provides restrictions on the exhibition and
training of performing animals. No person shall exhibit or train— any performing animal unless he is
registered in accordance with the provisions of this Chapter; as a performing animal, any animal which
the Central Government may, by notification in the Official Gazette, specify as an animal which shall not
be exhibited or trained as a performing animal. So, Statement 4 is correct.

83. Consider the following statements with reference to Coal gas :


1. Coal gas is formed by destructive distillation, which is the heating the bituminous coal in the
presence of air.
2. Coal gas is a mixture of Hydrogen, Carbon monoxide, and Methane.
Which of the statements given above is/are correct ?
(a) 1 only
(b) 2 only
87
(c) Both 1 and 2
(d) Neither 1 nor 2
EXPLANATION:
The process of heating coal in the absence of air is called the destructive distillation of coal. Coal contains
a number of elements, such as carbon, hydrogen, oxygen, nitrogen and sulphur. Several products are
obtained when coal is heated in the absence of air. The main products obtained by the destructive
distillation of coal are coke, coal tar, and coal gas. So, Statement 1 is not correct.
Coal gas is a gaseous mixture mainly hydrogen, methane, and carbon monoxide formed by the destructive
distillation of bituminous coal and used as a fuel. Sometimes steam is added to react with the hot coke,
thus increasing the gas yield.
The gases present in coal gas are combustible, making it an excellent fuel. It has a high calorific value. It
was used for lighting houses, factories, and streets in Mumbai until 1950. It was also used for cooling
earlier. So, Statement 2 is correct.

84. With respect to Indicator Species, consider the following statements :


1. Decreasing population of Lichen species is an indicator of Nitrogen pollution.
2. Decline in the Salmon population indicates an unclean and not well-connected river ecosystem.
3. Decline in the Rock Agama Lizards population size always indicates an increase in temperature in
the environment.
4. Presence of E.Coli in freshwater is an indicator of faecal contamination.
Which of the statements given above are correct ?
(a) 1, 3 and 4 only
(b) 2, 3 and 4 only
(c) 1, 2 and 4 only
(d) 1, 2, 3 and 4
EXPLANATION:
An indicator species is a species whose presence indicates the presence of a set of other species and whose
absence indicates the lack of that entire set of species. An indicator species is any biological species that
defines a trait or characteristic of the environment. A conservation practitioner can use an indicator
species as a surrogate for overall biodiversity, monitoring the outcomes of management practices by
measuring the rise or fall of the population of the indicator species.
 Lichens are a symbiotic relationship between algae and fungi. The fungus provides shelter for the
algae, and the algae provide food for the fungi. Lichens do not have roots; instead, they receive all their
nutrients from the atmosphere.
 Lichens are sensitive to atmospheric pollution such as nitrogen (N) because they receive all their
nutrients and water from wet and dry atmospheric deposition (fallout). Nitrogen deposition can
increase the load of nutrients. Too much N can harm and kill the algae's chlorophyll, which is used to
produce sugars feeding it and the fungi.

88
 Certain species of lichen are more tolerant of N than others. Scientists monitor lichen communities. If
an increase in N-tolerant species in combination with a decrease in N-sensitive species occurs, this
may indicate an increase in atmospheric N deposition.
So, Statement 1 is correct.
Salmon have a complex life history with many life stages occupying different habitats. A healthy salmon
population requires many different intact habitat types. Salmon is often referred to as an ecological
indicator species of wetlands because their success requires a wide variety of healthy habitats. When a
river ecosystem is not clean or well-connected, its salmon population will usually decline.
So, Statement 2 is correct.
The "Rock Agama Lizards' are found mainly in rocky places and warm spots. They do not generate their
own body heat, so they need to seek warmth from external sources, like a warm rock or a sunny spot.
They are important in the ecological aspect, as they can indicate which parts of the city are warming. In
warmer temperatures, their reproductive rate may increase, leading to a higher population. Conversely,
in cooler temperatures, their reproductive rate may decrease, which can lead to a lower population.
So, Statement 3 is not correct.
The concentration of faecal coliforms E. coli in freshwater bodies is an indirect indicator of contamination
with human and animal excreta.
Water contaminated with human and animal excreta poses a serious health risk and is therefore
unsuitable for potable supply unless it has been suitably treated. Faecal indicator bacteria, such as E.
coli, remain the preferred way of assessing the hygienic quality of water.
So, Statement 4 is correct.

85. Consider the following pairs :


Plateau Country
1. Potohar Plateau : Pakistan
2. Mascarene Plateau : Indian Ocean
3. Anatolia Plateau : Saudi Arabia
4. Karbi Anglong Plateau : Myanmar
How many pairs given above is/are correctly matched ?
(a) Only one pair
(b) Only two pairs
(c) Only three pairs
(d) All four pairs
EXPLANATION:
Potwar Plateau, tableland in Rāwalpindi, Attock, and Jhelum districts, Punjab Province, Pakistan. Lying
between the Indus and Jhelum rivers and bounded on the north by the Hazāra Hills and on the south by
the Salt Range, its varied landscape is constantly affected by erosion. The Potwar Plateau is one of the

89
most densely populated areas of Pakistan. The plateau is the location of Pakistan's major oil fields, the
first discovered at Khaur (1915) and Dhuliān (1935); the Tut field was discovered in 1968, and exploration
continued in the area in the 1970s. The oil fields are connected by pipeline to the refinery at Rawalpindi.
So, Pair 1 is correct.
Seychelles-Mauritius Plateau, also called Mascarene Plateau, is a submarine plateau made up of a very
shallow, extensive ridge in the Indian Ocean that forms a crescent through the Seychelles and Amirante
islands. The ridge extends from latitude 4° to 21° S and from longitude 54° to 63° E. It is believed to be a
small continental outlier similar to Madagascar and separated from the continent. The granitic Seychelles
islands rise from the ridge and are considered abnormal in an ocean basin because most oceanic rocks
are basaltic. The southern part of the plateau is, however, volcanic. So, Pair 2 is correct.

The Anatolian Plateau is a plateau that occupies most of Turkey's surface area. The central massif is
located between the Pontic and Taurus systems in the western half of the country. This elevated zone is
often referred to as the Anatolian plateau. The most distinctive part of the central massif is the area
bounded on the south by the Taurus Mountains and on the northeast by a line from Ankara through Lake
Tuz to Nigde. It is not located in Saudi Arabia. So, Pair 3 is not correct.

90
The Karbi Anglong District is situated in the central part of Assam. The district with dense tropical forest-
covered hills and flat plains. Karbi Anglong plateau is an extension of the Indian peninsular plateau in
Assam, the North Eastern state of India. This area receives maximum rainfall from the Southwest summer
Monsoon from June to September. It links with Meghalaya Plateau towards the south through a patch of
uneven terrain. So, Pair 4 is not correct.

86. Which one of the following straits connects the Sea of Azov with the Black Sea ?
(a) Strait of Hormuz
(b) Bosphorus Strait
(c) Kerch Strait
(d) Davis Strait
EXPLANATION:

91
The Kerch Strait is a strait in Eastern Europe. It is the only water body that connects the Black Sea and
the Sea of Azov, separating the Kerch Peninsula of Crimea in the west from the Taman Peninsula of
Russia's Krasnodar Krai in the east. So, Option (c) is correct.

87. Which of the following statements is/are correct about Temperature Inversion ?
1. Temperature inversion occurs when the normal decrease in temperature with altitude is reversed.
2. Clear Skies and light winds create conducive conditions for temperature inversion.
3. It can lead to the formation of smog and reduce air quality.
4. Temperature inversion is beneficial for the growth of cotton.
Select the correct answer using the code given below :
(a) 1 and 2 only
(b) 1, 2 and 3 only
(c) 2 and 4 only
(d) 1, 2, 3 and 4
EXPLANATION:
Long winter night, clear sky, dry air and the absence of winds leads to quick radiation of heat from the
Earth's surface and the lower layers of the atmosphere. This results in the cooling of the air near the
Earth's surface. The upper layers, which lose their heat not so quickly, are comparatively warm.
Hence the normal condition, in which temperature decreases with increasing height, is reversed. The
cooler air is nearer the Earth, and the warmer air is aloft. In other words, The phenomenon in which
temperature increases with increasing altitude temporarily and locally under certain conditions is known
as inversion of temperature.
So, Statements 1 and 2 are correct.
A thermal inversion will cause air quality to decrease because of the warmer "cap" layer that overlies the
surface air. This prevents convection from occurring and will cause any pollutants to be trapped near the
surface. Smog formation is an indicator of an inversion causing low air quality.
So, Statement 3 is correct.

92
Under normal conditions, the sun heats the Earth's surface. Warm air by the surface rises, and wind
allows for mixing warm air and the cooler air above it (temperature decreases with increasing altitude).
A temperature inversion is an exact opposite. This is when temperature increases as altitude increases.
In this situation, as the sun sets in the evening, radiative cooling of the Earth's surface occurs. This,
coupled with a lack of wind to facilitate mixing, cause a warm air layer to be trapped between cooler air
near the surface and cold air at higher altitudes. This essentially creates a ceiling where smog, for example,
can accumulate.
During normal conditions, the pesticides used in the farm area will dissipate. Under a temperature
inversion, spray solutions and smoke have the potential to move great distances offsite instead of
dissipating and diluting under normal atmospheric conditions. Therefore, a temperature inversion is
beneficial for growing cotton and other crops. So, Statement 4 is correct.

88. Which of the following is/are the characteristics of the Mediterranean climate ?
1. Cold, dry winters and warm, wet summers with onshore trade winds
2. Confined to the Western margins of the continents, between 30 to 40 degrees latitudes.
3. Hot, Dry summers and Mild, wet winters with onshore westerlies
Select the correct answer using the code given below :
(a) 1 only
(b) 2 and 3 only
(c) 2 only
(d) 3 only
EXPLANATION:
The Mediterranean type of climate is also known as the subtropical dry summer climate because most
areas falling under this climate are situated around the Mediterranean Sea. The Mediterranean climate
has three distinct characteristics
 A dry, warm summer with offshore trade winds.
 Bright, sunny weather with hot, dry summers and wet, mild winters.
 A concentration of rainfall in winter with onshore Westerlies.
 The prominence of local winds around the Mediterranean Sea.

93
The Mediterranean climate has developed between 30 to 40-degree latitudes in both the hemispheres in
the western parts of the continents.
Various kinds of vegetation are found in the Mediterranean regions, like,
 Mediterranean evergreen forests – open woodland with evergreen oaks, Eucalyptus forests (Australia).
 Evergreen coniferous trees – Pines, firs, cedars and cypresses
 Mediterranean bushes and shrubs.
 Conditions in the Mediterranean do not suit grass because most rain comes in the cool season when
growth is slow.
Economic development in these regions is Orchard farming, Crop cultivation and Wine production (85%
of grapes are produced in this region). So, Option (b) is correct.

89. With reference to Tropical cyclones, consider the following statements :


1. Tropical cyclones generally do not form at the Equator.
2. The wind speed is maximum at the eye of the cyclone.
3. Tropical cyclones develop only over oceans, while temperate cyclones develop over both land and
ocean.
Which of the statements given above is/are correct ?
(a) 1 and 2 only
(b) 3 only
(c) 1 and 3 only
(d) 1, 2 and 3

94
EXPLANATION:
Tropical cyclones are violent storms originating over oceans in tropical areas and moving to the coastal
areas, bringing about large-scale destruction caused by violent winds, heavy rainfall, and storm surges.
Tropical cyclones originate and intensify over warm tropical oceans. The conditions favourable for the
formation and intensification of tropical storms are:
 Large sea surface with a temperature higher than 27° C;
 Presence of the Coriolis force;
 Small variations in the vertical wind speed;
 A pre-existing weak low-pressure area or low-level-cyclonic circulation;
 Upper divergence above the sea level system.
Coriolis force is essential to provide the needed rotation. For e.g. At the Equator, the Coriolis force is zero
and the wind blows perpendicular to the isobars. The low pressure gets filled instead of getting intensified.
That is the reason why tropical cyclones are not formed near the Equator. So, Statement 1 is correct.

A mature tropical cyclone is characterized by the strong spirally circulating wind around the centre, called
the eye. The diameter of the circulating system can vary between 150 and 250 km. The eye is a region of
calm with subsiding air. Around the eye is the eye wall, where there is a strong spiraling ascent of air to
a greater height reaching the tropopause. The wind reaches maximum velocity in this region, reaching as
high as 250 km per hour. The wind speed is maximum at the eye wall. So, Statement 2 is not correct.
A tropical cyclone is a rapidly rotating storm originating over tropical oceans with more than 25 degrees
Celsius from where it draws the energy to develop. The tropical cyclones are strictly oceanic phenomenon.
The source of energy for tropical cyclone is warm oceanic water which provides moisture from sea on a
continuous basis. In the land the continuous supply of moisture is not available so tropical cyclones are
not formed over land. Temperate cyclones, also called extratropical cyclones, are atmospheric
disturbances with low pressure in the centre and increasing pressure outward. These cyclones can form
both in the land and the oceans, mainly formed in regions extending between 3 to 65-degree latitudes in
both hemispheres. So, Statement 3 is correct.

90. Consider the following statements :

95
1. Self-ploughing is a phenomenon associated with it.
2. They are formed by weathering and denudation of volcanic rocks.
3. It can retain moisture for a longer period.
Which type of soil is described in the above statements ?
(a) Black Soil
(b) Laterite Soil
(c) Peat Soil
(d) Arid Soil
EXPLANATION:
 Black soils are formed due to the weathering and denudation of volcanic rocks, that is, the
solidification of lava after volcanic eruptions.
 Black soil covers most of the Deccan Plateau, including parts of Maharashtra, Madhya Pradesh,
Gujarat, Andhra Pradesh and some parts of Tamil Nadu.
 The black soil is very deep in the upper reaches of the Godavari and the Krishna and the north western
part of the Deccan Plateau. These soils are also known as the ‘Regur Soil’ or the ‘Black Cotton Soil.’
 The black soils are generally clayey, deep and impermeable. They swell and become sticky when wet
and shrink when dried. So, during the dry season, these soils develop wide cracks. Thus, there occurs
a kind of ‘self-ploughing.’
 Because of this character of slow absorption and loss of moisture, the black soil retains the moisture
for a very long time, which helps the crops, especially; the rain-fed ones, to sustain even during the
dry season.
 Chemically, the black soils are rich in lime, iron, magnesia and alumina. They also contain potash.
But they lack phosphorous, nitrogen and organic matter. The colour of the soil ranges from deep black
to grey.
So, Option (a) is correct.

96
91. The term' Galathea Bay,' sometimes seen in the news recently, is related to :
(a) An important resort tourist destination of the Mediterranean region affected by rising sea levels.
(b) A place where marine organisms' death was reported due to heavy metal poisoning.
(c) A region where a large reserve of oil and natural gas was discovered in the Antarctica region.
(d) A breeding place for giant leatherback turtles is listed as vulnerable species in the IUCN Red
list.
EXPLANATION:
Galathea Bay is at Great Nicobar Island in the Andaman and Nicobar Island Chain. Galathea National
Park has an extreme tropical climate and high humidity levels and is made up of lowland forests with the
highest concentration of indigenous avifauna. Galathea Bay is a well-known breeding place for the
mysterious giant leatherback, the world's largest marine turtle, which is listed as vulnerable species in

97
the IUCN Red list. Also, it includes the critically endangered Great Nicobar crake, the Nicobar frog, the
Nicobar shrew, the Nicobar cat snake, and a new lizard. So, Option (d) is correct.

92. With respect to Oceanic tides, consider the following statements :


1. Neap tides occur during the full moon and new moon days.
2. Spring Tides occur when the sun and the moon are at right angles to each other.
3. The tidal range is usually higher at perigee than apogee.
Which of the statements given above is/are not correct ?
(a) 1 and 2 only
(b) 2 and 3 only
(c) 1 and 3 only
(d) 1, 2 and 3
EXPLANATION:
Average tidal ranges are slightly larger during full or new moons, which occur when the Earth, sun, and
moon are nearly in alignment. This occurs twice each month. The moon appears new (dark) when directly
between the Earth and the sun.
The moon appears full when the Earth is between the moon and the sun. In both cases, the sun's
gravitational pull is "added" to the gravitational pull of the moon on Earth, causing the oceans to bulge a
bit more than usual. This means that high tides are slightly higher, and low tides are slightly lower than
average.

98
These are called spring tides. The term is derived from the concept of the tide "springing forth." Spring
tides occur twice each lunar month all year long, without regard to the season.
So, Statement 1 is not correct.
Neap tide occurs after Seven days of a spring tide, when the sun and moon are at right angles to each
other. When this happens, the bulge of the ocean caused by the sun partially cancels out the bulge of the
ocean caused by the moon. This produces moderate tides, known as neap tides, meaning that high tides
are slightly lower and low tides are slightly higher than average. Neap tides occur during the first and
third quarter moon when the moon appears "half full." So, Statement 2 is not correct.

Once a month, when the moon is closest to the Earth (at perigee), tide-generating forces are higher than
usual, producing above-average ranges in the tides. About two weeks later, the lunar tide-raising force is
smaller when the moon is farthest from the Earth (at apogee), and the tidal ranges are less than average.
So, Statement 3 is correct.

93. Consider the following pairs :


National Parks Rivers associated with it
1. Orang National Park - River Dhanshri
2. Ranthambore National Park - River Chambal
3. Silent Valley National Park - River Bhavani
How many pairs given above is/are correctly matched ?
(a) Only one pair
(b) Only two pairs
(c) All of the pairs
(d) None of the pairs
EXPLANATION:
Orang National Park is the oldest game reserve of the state, just on the northern bank of the river
Brahmaputra with an area of 78.80 sq. km., and it is an important breeding ground for varieties of fish.
The Park is situated on the northern bank of the Brahmaputra River. The Pachnoi and Dhansiri rivers

99
flow along its eastern and western boundaries, respectively. Both of these rivers are tributaries of the
Brahmaputra.
It is located on the north bank of the Brahmaputra River in the Darrang and Sonitpur districts of Assam,
India. It was established as a sanctuary in 1985 and declared a national park on 13 April 1999. It is also
known as the mini Kaziranga National Park (IUCN site) since the two parks have a similar landscape made
up of marshes, streams, and grasslands and are inhabited by the Great Indian one-horned rhinoceros.
The Park has rich flora and fauna, including the great Indian one-horned rhinoceros, pigmy hogs,
elephants, wild buffalo, and tigers. It is the only stronghold of the rhinos on the north bank of the
Brahmaputra River. So, pair 1 is correct.
Ranthambore National Park is one of the largest national parks in northern India. It is situated 14
kilometers from Sawai Madhopur district and about 180 kilometers from Jaipur in southeastern
Rajasthan in India.
The vegetation of the Ranthambhore is the tropical dry deciduous and tropical thorn types due to its hilly
terrain; water is confined to narrow valleys and some lakes. There are several lakes in the Park: Padam
Talab, Raj Bagh Talab, and Malik Talab. It is bounded to the north by the Banas River and to the south
by the Chambal River. It is named after the historic Ranthambore Fort within the Park. So, Pair 2 is
correct.
Silent Valley National Park is a beautiful representation of the last remaining rainforest in Kerala. A
perennial river named Kunthipuzha passes through the western side of the Park, from north to south,
finally merging into Bharathapuzha.
The main tributaries of this river are Kunthancholapuzha, Karingathodu, Madrimaranthode,
Valiaparathodu, and Kummathanthode. All major tributaries of Kunthipuzha originate on the upper
slopes of the eastern side of the valley. The streams from the western slopes are dry in summer.
So, Pair 3 is not correct.

94. Wildfires are predicted to worsen in the coming years and decades, as per United Nations Environment
Programme's (UNEP) Annual Frontiers Report 2022. In this context, consider the following statements
:
1. Wildfires are generally a natural phenomenon.
2. Extreme weather events can lead to longer fire seasons and increase the likelihood of fire weather
conditions.
3. Lightning ignition is the predominant driver of massive wildfires in boreal forests.
4. Higher frequency of forest fires can result in more frequent incidences of lightning.
Which of the statements given above is/are correct?
(a) 1 only
(b) 2 and 3 only
(c) 1, 2 and 3 only
(d) 1, 2, 3 and 4
EXPLANATION:
100
According to the United Nations Environment Programme (UNEP) annual Frontiers Report 2022,
 A wildfire is an unplanned fire that burns in a natural area such as a forest, grassland, or prairie.
Wildfires are a natural phenomenon but are becoming more dangerous and affecting larger areas. The
UN report has attributed this to climate change and human activities.
 The trends towards more dangerous fire-weather conditions are likely to increase due to rising
concentrations of atmospheric greenhouse gases and the attendant escalation of wildfire risk factors.
 Extreme weather events such as hotter temperatures and more droughts lead to longer fire seasons
and increase the likelihood of fire weather conditions.
 Wildfires have also become more common in the Savannah ecosystem. Savannah, or the mixed
woodland-grassland ecosystem, accounted for 77 percent of the total 13 million individual fires (lasting
4-5 days) globally between 2003 and 2016. This has affected over one-fourth of the species in the
Savannah ecosystem.
 With rising forest fires, the world is very likely to see more frequent incidences of lightning. Lightning
strikes are projected to increase in frequency in some parts of the world as the climate changes.
Lightning can spark wildfires. But wildfires can also spark lightning. Many factors are believed to
cause lightning during a wildfire. Scientists have blamed everything from air currents to urbanization.
These elements have been hard to disentangle in the air above land because the rough topography,
changing land use, and varying heat create a complex set of variables that are hard to tease out. But,
Lightning causes wildfires and wildfires enhance lightning.
 Lightning ignition is the predominant driver of massive wildfires in the boreal forests of North America
and northern Siberia.
 In 2021, Africa was the most affected region due to wildfires.
 The report also recommended the following to improve the monitoring and management of wildfires:
 Appreciating and adopting indigenous fire management techniques.
 Focus on long-range weather forecasting.
 Focus on remote-sensing capabilities such as satellites, ground-based radar, lightning detection,
and data handling.
So, Option (d) is correct.

95. Which of the following statements best describes ‘Biostimulants’ in agriculture ?


(a) Biostimulants are biological substances that enhance crop yield by releasing nutrient content into
the Soil.
(b) Biostimulants are microorganisms that live in the Soil and help plants absorb nutrients and grow
well.
(c) Biostimulants are substances or microorganisms applied to plants, soils or seeds that
improve plant growth and development.
(d) Biostimulants are bio-pesticides that protect crops from diseases and pests.
EXPLANATION:

101
Biostimulants mean compounds, substances and products, including microorganisms whose functions,
when applied to plants/seeds/ rhizosphere is, to regulate and enhance a crop's physiological processes
independent of the product's nutrient contents to improve input use efficiency, growth, yield, quality
and/or stress tolerance. The bio-stimulants may include products of plants /animals or microbial origin.
Bio Stimulants shall be under any of the following categories, namely,
 Botanical extracts, including seaweed extracts
 Biochemicals
 Protein hydrolysates and amino acids
 Vitamins
 Cell-free Microbial products
 Antioxidants
 Anti-transpirants
 Humic & fulvic acid and their derivations
Biostimulants are products that reduce the need for fertilizers and increase plant growth and resistance
to water and abiotic stresses. In small concentrations, these substances are efficient, favoring the plant's
vital processes' good performance and allowing high yields and good-quality products. In addition,
biostimulants applied to plants enhance nutrition efficiency, abiotic stress tolerance and/or plant quality
traits, regardless of their nutrient contents. So, Option (c) is correct.

96. With reference to Migratory flyways, consider the following statements :


1. The Bonn convention is the only global convention specializing in the conservation of migratory
species, their habitats and migration routes.
2. Typically, migration follows the east–west axis, with birds moving to milder climates at lower
latitudes for the duration of their non-breeding season.
3. There are 8 Global Flyways, among which 3 are in India
4. The State of Gujarat, Rajasthan, Uttar Pradesh, and Maharashtra falls under the East Asian
flyways.
Choose the correct answer using the code given below :
(a) 1, 3 and 4 only
(b) 1 and 3 only
(c) 2 and 4 only
(d) 1, 2, 3 and 4
EXPLANATION:
The Convention on Migratory Species (CMS), also known as the Bonn Convention, is an environmental
treaty of the United Nations that provides a global platform for the conservation and sustainable use of
terrestrial, aquatic, and avian migratory animals and their habitats.
 As the only global convention specializing in the conservation of migratory species, their habitats,
and migration routes, CMS complements and cooperates with a number of other international
organizations, NGOs, and partners in the media as well as in the corporate sector.

102
So, Statement 1 is correct.
The flyway concept is an important tool for galvanizing international conservation action. Typically,
migration follows a north–south axis, with birds moving to milder climes at lower latitudes for the duration
of their non-breeding season (Newton 2008).
The energy costs associated with migration mean that birds are under pressure to use the shortest possible
route; however, the precise course taken will depend on weather patterns, en-route resources, and the
geographical features they encounter. Thus, routes often follow mountain ranges, watercourses and
coastlines, avoid large bodies of open water and take advantage of prevailing wind patterns and updrafts.
So, Statement 2 is not correct.
A flyway is a geographical region within which a single or a group of migratory species completes its annual
cycle: breeding, moulting, staging, and non-breeding.
There are eight flyways in the world. Three of the eight flyways pass through the Indian Subcontinent.
 The Central Asian Flyway (CAF), East Asian Australasian Flyway, and Asian East African Flyway are
the three flyways that pass through India. So, Statement 3 is correct.

The East Asia/Australasia Flyway extends from Arctic Russia and North America to the southern limits of
Australia and New Zealand. It encompasses large parts of East Asia, all of Southeast Asia, and includes
eastern India, and the Andaman and Nicobar Islands. So, Statement 4 is not correct.

97. Barnase/Barstar system is mentioned in the news is related to :


(a) Encryption in Blockchain Technology
(b) Terminator seed technology
(c) Precision farming technology
(d) Genetically modified Mustard
EXPLANATION:
Recently, Approval was given for the environmental release of Genetically Modified (GM) mustard DMH 11
by the Genetic Engineering Appraisal Committee (GEAC) under the Union Ministry of Environment, Forest
and Climate Change.
Genetically Modified technology is a disruptive technology capable of bringing any targeted change within

103
the crop variety to overcome a difficult problem that is also safe for humans, animals and the environment.
The barnase-barstar system is an indelible technique to produce hybrid seeds in self-pollinated crops. The
genetically engineered barnase/barstar system provides an efficient and robust alternative method for
hybrid seed production in Mustard. It has been successfully deployed in Canada, Australia and America
for decades. Hybrid technology can play an important role in enhancing the productivity of rapeseed
mustard in the country.
In India, the Centre for Genetic Manipulation of Crop Plants (CGMCP), the University of Delhi South
Campus, New Delhi, has made a successful attempt with some alterations in the barnase/ barstar system,
which culminated in the development of GM Mustard hybrid MH11 which has undergone the required
regulatory testing processes during 2008-2016.
So, Option (d) is correct.

98. Which of the following statements regarding the International Criminal Court (ICC) is not correct ?
(a) ICC was established to fight global impunity and bring to justice criminals under international
law, regardless of their rank or stature.
(b) As a judicial institution, the ICC has its own police force and enforcement body.
(c) ICC is governed by an international treaty called the Rome Statute.
(d) The Court has no power to arrest sitting heads of State or bring them to trial.

EXPLANATION:
The International Criminal Court (ICC) investigates and, where warranted, tries individuals charged with
the gravest crimes of concern to the international community: genocide, war crimes, crimes against
humanity and the crime of aggression.
The Court is participating in a global fight to end impunity. Through international criminal justice, the
Court aims to hold those responsible accountable for their crimes and help prevent them from happening
again.
The International Criminal Court is not a substitute for national courts. According to the Rome Statute,
every State has to exercise its criminal jurisdiction over those responsible for international crimes.
So, Option (a) is correct.
As a judicial institution, the ICC has no police force or enforcement body. Thus, it relies on cooperation
with countries worldwide for support, particularly for making arrests, transferring arrested persons to the
ICC detention center in The Hague, freezing suspects’ assets, and enforcing sentences.
While not a United Nations organization, the Court has a cooperation agreement with the United Nations.
When a situation is not within the Court’s jurisdiction, the United Nations Security Council can refer it to
the ICC, granting it jurisdiction.
So, Option (b) is not correct.

104
On 17 July 1998, a conference of 160 States established the first treaty-based permanent international
criminal Court. The treaty adopted during that conference is the Rome Statute of the International
Criminal Court.
Among other things, it sets out the crimes falling within the jurisdiction of the ICC, the rules of procedure
and the mechanisms for States to cooperate with the ICC. The countries which have accepted these rules
are known as States Parties and are represented in the Assembly of States Parties.
The Assembly of States Parties, which meets at least once a year, sets the general policies for the
administration of the Court and reviews its activities. So, Option (c) is correct.
The Court has no power to arrest sitting heads of State or bring them to trial and instead must rely on
other leaders and governments to act as its sheriffs around the world. A suspect who manages to evade
capture may never have a hearing to confirm the charges. So, Option (d) is correct.

99. With reference to Conference on Interaction and Confidence Building Measures in Asia (CICA),
consider the following statements :
1. It is an inter-governmental forum aimed at promoting peace, security and stability in Asia.
2. Currently, India has an observer status in this forum.
3. The third conference on 'No Money for Terrorism', which is an initiative of CICA, was held recently
in 2022.
Which of the statements given above is/are correct ?
(a) 1 only
(b) 2 only
(c) 2 and 3 only
(d) 1, 2 and 3
EXPLANATION:
The Conference on Interaction and Confidence Building Measures in Asia (CICA) is a
international forum for enhancing cooperation towards promoting peace, security and stability in Asia. It
is a forum based on the recognition that there is a close link between peace, security and stability
in Asia and the rest of the world.
The idea of convening CICA was first proposed by the First President of the Republic of Kazakhstan -
Elbasy H.E., Mr. Nursultan Nazarbayev, on 5 October 1992, at the 47th Session of the United Nations
General Assembly. The moving spirit behind this initiative was the aspiration to set up an efficient and
acceptable structure for ensuring peace and security in Asia. The CICA Secretariat has been in Almaty
(Kazakhstan) since June 2006. So, Statement 1 is correct.
The Conference on Interaction and Confidence Building Measures (CICA) members include 27 Asian
countries, including Azerbaijan, Bahrain, China, Egypt, India, Iran, Israel, Russia, South Korea, and
Turkey, nine observer states, and five international organizations. India has been a member right since
CICA's inception.

105
The Observer states Belarus, Indonesia, Japan, Laos, Malaysia, Philippines, Turkmenistan, Ukraine and
the USA.
India has offered to co-coordinate CBMs on Energy Security with the Republic of Korea and CBMs in
developing secure and effective transportation corridors with Azerbaijan.
So, Statement 2 is not correct.
Recently, the third 'No Money for Terror' (NMFT) Ministerial Conference on Counter-Terrorism
Financing was held in New Delhi, India. The honourable Prime Minister of India has strongly asked to
avoid any ambiguity in dealing with terrorism and also warned against nations that use terrorism as a
tool of foreign policy.
“No Money for Terrorism” is an initiative launched by the Financial Action Task Force (FATF), an
intergovernmental organization that sets standards and promotes the implementation of measures to
combat money laundering, terrorist financing, and other threats to the integrity of the international
financial system. So, Statement 3 is not correct.

100. With respect to World Trade Organisation (WTO), consider the following statements :
1. The Ministerial Conference is the WTO's top decision-making body and meets annually.
2. To protect global fish stocks better, WTO passed a multilateral agreement to curb 'harmful'
subsidies on illegal, unreported and unregulated fishing for the next four years.
3. WTO's Agreement on Government Procurement is to mutually open government procurement
markets among its parties which is binding in nature.
Which of the statements given above is/are correct ?
(a) 1 only
(b) 2 only
(c) 2 and 3 only
(d) 1 and 3 only
EXPLANATION:
The World Trade Organization (WTO) is the only global international organization dealing with trade rules
between nations. The Ministerial Conference is the topmost body of the WTO under the governance
structure set up by the "Agreement establishing the WTO."
The Ministerial Conference, which usually meets every two years, brings together all members of the WTO,
all of which are countries or customs unions. The Ministerial Conference can decide on all matters under
any multilateral trade agreement. So, Statement 1 is not correct.
The WTO Agreement on Fisheries Subsidies, adopted at the 12th Ministerial Conference (MC12) on 17
June 2022, marks a major step forward for ocean sustainability by prohibiting harmful fisheries subsidies,
which are a key factor in the widespread depletion of the world's fish stocks.
 MC12 is the first broad, binding, multilateral Aogreement on ocean sustainability and only the second
Agreement reached at the WTO since its inception. For the Agreement to become operational, two-
thirds of members have to deposit their "instruments of acceptance" with the WTO.

106
 As per Article 25 of the SCM Agreement, If comprehensive disciplines are not adopted within four years
of the entry into force of this Agreement by the member nation, and unless otherwise decided by the
General Council, this Agreement shall stand immediately terminated. Thus, this MC12 Agreement is
valid for the next four years.
So, Statement 2 is correct.
To ensure open, fair and transparent conditions of competition in the government procurement markets,
some WTO members have negotiated the Agreement on Government Procurement (GPA).
Government Procurement (GPA) is a plurilateral agreement within the framework of the WTO, meaning
that not all WTO members are parties to the Agreement. At present, the Agreement has 21 parties
comprising 48 WTO members.
The fundamental aim of the GPA is to mutually open government procurement markets among its parties.
As a binding international treaty, the GPA is administered by the Committee on Government Procurement
which is composed of representatives of all its parties. The enforcement of the Agreement is realized
through two mechanisms: the domestic review mechanism at the national level and the WTO dispute
settlement mechanism at the international level. So, Statement 3 is correct.

107

You might also like